Pediatric Nursing Study Questions

Pataasin ang iyong marka sa homework at exams ngayon gamit ang Quizwiz!

The clinic nurse completed a teaching session with a 16-year-old patient who is being treated for acne vulgaris. Which statement indicates understanding of the teaching points? 1) "I will use a gentle cleanser once or twice daily." 2) "I will clean my face twice daily with a facial scrub pad." 3) "I will use the most expensive cleanser on the market to make sure I am getting the best product available." 4) "I will squeeze the whitehead to keep the pores open."

1) "I will use a gentle cleanser once or twice daily." Rationale: Gentle cleansing with a mild cleanser once or twice daily is usually sufficient. Adolescents need education about the factors that aggravate and damage the skin such as too vigorous scrubbing, squeezing and manual expression with the fingernails. Information to dispel myths regarding products on the market can prevent unnecessary costs.

Which comment made by a 12-year-old child with insulin-dependent diabetes mellitus (IDDM) indicates a knowledge deficit? 1) "I'll be glad when I can take a pill for my diabetes like my uncle does." 2) "I keep Lifesavers in my school bag in case I have a low-sugar reaction." 3) "I rotate my insulin injection sites every time I give myself an injection." 4) "I keep records of my glucose levels and insulin sites and amounts."

1) "I'll be glad when I can take a pill for my diabetes like my uncle does." Rationale: Type 1 diabetes results when the pancreas is unable to produce and secrete an adequate amount of insulin. This form of diabetes, the most common childhood endocrine disorder, requires lifelong therapeutic management with insulin therapy.

Which statement by the nurse is most appropriate to a 15-year-old adolescent whose friend has mentioned suicide? 1) "Your friend's threat needs to be taken seriously and immediate help for your friend is important." 2) "If your friend mentions suicide a second time, you will want to get your friend some help." 3) "Tell your friend to come to the clinic immediately." 4) "You need to gather details about your friend's suicide plan."

1) "Your friend's threat needs to be taken seriously and immediate help for your friend is important." Rationale: Nurses should counsel clients that all adolescent suicidal threats and gestures should be taken very seriously. Adolescents identified as at risk for suicide and their families should be targeted for supportive guidance and counseling before a crisis situation.

Pertussis vaccination should begin at which age? 1) 2 months 2) 12 months 3) Birth 4) 6 months

1) 2 months Rationale: The acellular pertussis vaccine is recommended by the American Academy of Pediatrics beginning at age 6 weeks. Infants are at greater risk for complications of pertussis. The vaccine is not given after age 7 years, when the risks of the vaccine become greater than those of pertussis. The first dose is usually given at the 2-month well-child visit. Infants are highly susceptible to pertussis, which can be a life-threatening illness in this age-group.

In term newborns, the first meconium stool typically occurs within the first ____ hr after birth. 1) 24 2) 8 3) 12 4) 6

1) 24 Rationale: The first meconium stool should occur within the first 24 hr. It may be delayed up to 7 days in very low-birth-weight newborns.

The nurse notes that an infant stares at an object placed in his hand and takes it to his mouth, coos and gurgles when talked to, and sustains part of his own weight when held in a standing position. The nurse correctly interprets these findings as characteristics of an infant at which age? 1) 4 months 2) 7 months 3) 2 months 4) 9 months

1) 4 months Rationale: Holding the head erect when sitting, staring at an object placed in the hand, taking the object to the mouth, cooing and gurgling, and sustaining part of his body weight when in a standing position are behaviors characteristic of a 4-month-old infant. A 2-month-old infant typically vocalizes, follows objects to midline, and smiles. A 7-month-old typically is able to sit without support, turns toward the voice, and transfers objects from hand to hand. Usually , a 9-month-old can crawl, stand while holding on, and initiate speech sounds.

A 2-year-old always puts his teddy bear at the head of his bed before he goes to sleep. The parents ask the nurse if this behavior is normal. The nurse should explain to the parents that toddlers use ritualistic patterns to establish which factor? 1) A sense of security 2) Control over adults in their environment 3) A sense of identity 4) Sequenced patterns of learning behavior

1) A sense of security Rationale: Toddlers establish ritualistic patterns to feel secure, despite inconsistencies in their environment. Establishing a sense of identity is the developmental task of the adolescent. The toddler's developmental task is to use rituals and routine to help in making autonomy easier to accomplish. Ritualistic patterns do involve patterns of behavior, but they are not utilized to develop learning behaviors.

A mother tells the nurse that she does not want her infant immunized because of the discomfort associated with injections. What should the nurse explain? 1) A topical anesthetic can be applied before injections are given. 2) Infants do not feel pain as adults do. 3) This cannot be prevented. 4) This is not a good reason for refusing immunizations.

1) A topical anesthetic can be applied before injections are given. Rationale: To minimize the discomfort associated with intramuscular injections, a topical anesthetic agent can be used on the injection site. These include EMLA (eutectic mixture of local anesthetic) and vapor coolant sprays. Pain associated with many procedures can be prevented or minimized by using the principles of atraumatic care. Infants have neural pathways that will indicate pain. Numerous research studies have indicated that infants perceive and react to pain in the same manner as do children and adults. The mother should be allowed to discuss her concerns and the alternatives available. This is part of the informed consent process.

The nurse is planning care for a patient with a different ethnic background. Which should be an appropriate goal? 1) Adapt, as necessary, ethnic practices to health needs. 2) Attempt, in a nonjudgmental way, to change ethnic beliefs. 3) Encourage continuation of ethnic practices in the hospital setting. 4) Strive to keep ethnic background from influencing health needs.

1) Adapt, as necessary, ethnic practices to health needs. Rationale: Whenever possible, nurses should facilitate the integration of ethnic practices into health care provision. The ethnic background is part of the individual; it should be difficult to eliminate the influence of ethnic background. The ethnic practices need to be evaluated within the context of the health care setting to determine whether they are conflicting.

A toddler has moderate respiratory distress, is mildly cyanotic, and has increased work of breathing, with a respiratory rate of 40. What is the priority nursing intervention? 1) Airway maintenance and 100% oxygen by mask. 2) Airway maintenance and continued reassessment. 3) 100% oxygen and provision of comfort. 4) 100% oxygen and pulse oximetry monitoring.

1) Airway maintenance and 100% oxygen by mask. Rationale: Priorities of care for the child with respiratory distress are to clear the airway and provide oxygen supplementation.

When the nurse interviews an adolescent, which is especially important? 1) Allow an opportunity to express feelings. 2) Focus the discussion on the peer group. 3) Use the same type of language as the adolescent. 4) Emphasize that confidentiality will always be maintained.

1) Allow an opportunity to express feelings. Rationale: Adolescents, like all children, need opportunities to express their feelings. Often, they interject feelings into their words. The nurse must be alert to the words and feelings expressed. The nurse should maintain a professional relationship with adolescents. To avoid misunderstanding or misinterpretation of words and phrases used, the nurse should clarify the terms used, what information will be shared with other members of the health care team, and any limits to confidentiality. Although the peer group is important to this age-group, the interview should focus on the adolescent.

The school nurse is explaining to older school children that obesity increases the risk for which disorders? (Select all that apply.) 1) Altered glucose metabolism 2) Hypertension 3) Asthma 4) Dyslipidemia 5) Irritable bowel disease

1) Altered glucose metabolism 2) Hypertension 4) Dyslipidemia Irritable bowel disease and asthma are not linked to obesity.

The nurse is caring for a child who has had diarrhea and vomiting for the past several days. What is the priority nursing assessment? 1) Assess circulation and perfusion 2) Determine the child's weight 3) Send a stool specimen to the lab 4) Ask if the family has traveled outside of the country

1) Assess circulation and perfusion Rationale: Infants and children are at increased risk for dehydration and hypovolemia compared with adults. The nurse must quickly determine if the child with diarrhea and vomiting needs rehydration to prevent further (and usually quick) deterioration.

A nurse is caring for a child who sustained a fracture. Which of the following actions should the nurse take? (Select all that apply.) 1) Assess neurovascular status frequently. 2) Place a heat pack on the site of the injury. 3) Stabilize the injury. 4) Elevate the affected limb. 5) Encourage ROM of the affected limb.

1) Assess neurovascular status frequently. 3) Stabilize the injury. 4) Elevate the affected limb.

The nurse is reviewing the importance of role learning for children. The nurse understands that children's roles are primarily shaped by which members? 1) Peers 2) Parents 3) Grandparents 4) Siblings

2) Parents Rationale: Children's roles are shaped primarily by the parents, who apply direct or indirect pressures to induce or force children into the desired patterns of behavior or direct their efforts toward modification of the role responses of the child on a mutually acceptable basis.

The nurse needs to assess a 15-month-old child who is sitting quietly on his father's lap. What initial action by the nurse would be most appropriate? 1) Begin the assessment while the child is in his father's lap. 2) Ask the father to place the child on the exam table. 3) Undress the child while he is still sitting on his father's lap. 4) Talk softly to the child while taking him from his father.

1) Begin the assessment while the child is in his father's lap. Rationale: For young children, particularly infants and toddlers, preserving parent-child contact is a good way of decreasing stress or the need for physical restraint during an assessment. For example, much of a patient's physical examination can be done with the patient in a parent's lap with the parent providing reassuring and comforting contact. The initial action would be to begin the assessment while the child is in his father's lap.

A child has been diagnosed with a Wilms tumor. What should preoperative nursing care include? 1) Careful bathing and handling 2) Monitoring of behavioral status 3) Administration of packed red blood cells 4) Maintenance of strict isolation

1) Careful bathing and handling Rationale: Careful bathing and handling are important in preventing trauma to the Wilms tumor site.

Which is the single most important factor to consider when communicating with children? 1) Child's developmental level 2) Presence of the child's parent 3) Child's nonverbal behaviors 4) Child's physical condition

1) Child's developmental level Rationale: The nurse must be aware of the child's developmental stage to engage in effective communication. The use of both verbal and nonverbal communication should be appropriate to the developmental level. Nonverbal behaviors vary in importance based on the child's developmental level and physical condition. Although the child's physical condition is a consideration, developmental level is much more important. The presence of parents is important when communicating with young children but may be detrimental when speaking with adolescents.

Additional biobehavioral interventions may be helpful in reducing pain. Which of the following may be considered? (Select all that apply.) 1) Distraction 2) Cognitive-behavioral therapy 3) Steroidal treatment 4) Transcutaneous nerve stimulation 5) Percutaneous nerve ablation

1) Distraction 2) Cognitive-behavioral therapy 4) Transcutaneous nerve stimulation Rationale: Biobehavioral interventions prevent and treat pain by interrupting the pain, fear, anxiety, and stress cycle. Distraction, relaxation, guided imagery, hypnosis, CBT, massage, heat, cold, and transcutaneous nerve stimulation can help with pain control. Steroids are a pharmacologic treatment for inflammatory disorders, and the percutaneous nerve ablation is a minimally invasive procedure often seen for lower lumbar disorders.

The nurse is preparing to admit a 1-year-old child with pertussis (whooping cough). Which clinical manifestations of pertussis should the nurse expect to observe? (Select all that apply.) 1) Dry hacking cough 2) Coryza 3) Conjunctivitis 4) Low-grade fever 5) Earache

1) Dry hacking cough 2) Coryza 4) Low-grade fever Rationale: The clinical manifestations of pertussis include coryza, a low-grade fever, and a dry hacking cough. The child does not have an earache or conjunctivitis.

A visitor to the hospital has a cardiac arrest. When determining to use an automated external defibrillator (AED), the nurse should consider the AEDs are used in cardiac arrest in which circumstances? 1) Early defibrillation in cases of ventricular fibrillation 2) Early defibrillation in cases of atrial fibrillation 3) Cardioversion in cases of atrial fibrillation 4) Pacemaker placement

1) Early defibrillation in cases of ventricular fibrillation Rationale: AEDs are used for early defibrillation in cases of ventricular fibrillation. The AHA and Canadian Heart and Stroke Foundation place major emphasis on early defibrillation for ventricular fibrillation and use of the AED as a tool to increase sudden cardiac arrest survival rates.

Which is the priority outcome of mandatory newborn screening for hypothyroidism? 1) Early identification of the disorder 2) Parental education about raising a special needs child 3) Appropriate community referral for affected families 4) Facilitation of parent-child bonding

1) Early identification of the disorder Rationale: Early identification of hypothyroidism is vital, because, if left untreated, it will result in irreversible mental retardation.

The nurse is aware that if patients' different cultures are implied to be inferior, the emotional attitude the nurse is displaying is what? 1) Ethnocentrism 2) Acculturation 3) Cultural shock 4) Cultural sensitivity

1) Ethnocentrism Rationale: Ethnocentrism is the belief that one's way of living and behaving is the best way. This includes the emotional attitude that the values, beliefs, and perceptions of one's ethnic group are superior to those of others. Acculturation is the gradual changes that are produced in a culture by the influence of another culture that cause one or both cultures to become more similar. The minority culture is forced to learn the majority culture to survive. Cultural shock is the helpless feeling and state of disorientation felt by an outsider attempting to adapt to a different culture group. Cultural sensitivity, a component of culturally competent care, is an awareness of cultural similarities and differences.

An 8-year-old girl is being admitted to the hospital from the emergency department with an injury from falling off her bicycle. What intervention will help her most in her adjustment to the hospital? 1) Explain hospital schedules to her, such as mealtimes. 2) Use terms such as "honey" and "dear" to show a caring attitude. 3) Orient her parents, because she is too young, to her room and hospital facility. 4) Explain when parents can visit and why siblings cannot come to see her.

1) Explain hospital schedules to her, such as mealtimes. Rationale: School-age children need to have control of their environment. The nurse should offer explanations or prepare the child for what to expect. The nurse should refer to the child by the preferred name. Explaining when parents can visit and why siblings cannot come focuses on the limitations rather than helping her adjust to the hospital. At the age of 8 years, the child should be oriented to the environment along with the parents.

What approach by the nurse would most likely encourage a child to cooperate with an assessment of physical and developmental health? 1) Explain what is going to happen in words the child can understand. 2) Force the child to cooperate by having a parent hold him or her down. 3) Give the child a sticker before beginning the examination. 4) Explain to the child what's going to happen when the child asks questions.

1) Explain what is going to happen in words the child can understand. Rationale: Explain briefly what is going to happen, using words the child can understand.

As part of the diagnostic evaluation of a child with cancer, biopsies are important for staging. What statement explains what staging means? 1) Extent of the disease at the time of diagnosis 2) Rate normal cells are being replaced by cancer cells 3) Biologic characteristics of the tumor or lymph nodes 4) Abnormal, unrestricted growth of cancer cells producing organ damage

1) Extent of the disease at the time of diagnosis Rationale: Staging is a description of the extent of the disease at the time of diagnosis. Staging criteria exist for most tumors. The stage usually relates directly to the prognosis; the higher the stage, the poorer the prognosis. The rate that normal cells are being replaced by cancer cells is not a definition of staging. Classification of the tumor refers to the biologic characteristics of the tumor or lymph nodes. Abnormal, unrestricted growth of cancer cells producing organ damage describes how cancer cells grow and can cause damage to an organ.

The nurse is preparing to assess the pain of a 3-year-old child who had surgery the day before. Which pain assessment method would be most appropriate for the nurse to use? 1) FACES pain rating scale, observation of the child, and parent report 2) Visual analog scale 3) Asking the parents to rate their child's pain using the word-graphic rating scale 4) FACES pain rating scale and poker chip tool

1) FACES pain rating scale, observation of the child, and parent report Rationale: The FACES pain rating scale is a self-report tool that can be used by children as young as 3 or 4 years of age. Self-report measures should be used in conjunction with observation and discussion with the child and family in children under 5.

A child is diagnosed with hypothyroidism. The nurse should expect to assess which symptoms associated with hypothyroidism? (Select all that apply.) 1) Fatigue 2) Cold intolerance 3) Weight loss 4) Dry, thick skin 5) Diarrhea

1) Fatigue 2) Cold intolerance 4) Dry, thick skin Rationale: S&S of hypothyroidism include: Large for age despite having poor feeding habits; increased birth weight Puffy face, swollen tongue Hoarse cry Poor muscle tone Cold extremities Persistent constipation; bloated or full to the touch Dry, thick skin Cold intolerance Lack of energy, sleeps most of the time, appears tired even when awake Little to no growth Often appears perfectly normal at birth

A mother brings her 6 month old infant to the clinic. The child has been vomiting since early morning and has had diarrhea since the day before. His temperature is 38 C, pulse 140, and respiratory rate 38. He has lost 6 oz since his well-child visit 4 days ago. He cries before passing a bowel movement. He will not breastfeed today. What is the priority nursing diagnosis? 1) Fluid volume deficit related to excessive losses and inadequate intake 2) Thermoregulation alteration 3) Pain (abdominal) related to diarrhea 4) Alteration in nutrition, less than body requirements, related to decreased oral intake

1) Fluid volume deficit related to excessive losses and inadequate intake Rationale: Infants are at significant risk for dehydration due to their increased proportion of body fluid as compared with adults. Although fever and pain are important, fluid volume takes priority in the infant with fluid losses or decreased intake.

A 10-year-old boy is admitted to the hospital with a diagnosis of appendicitis. He is nauseated, febrile, and complaining of severe abdominal pain radiating to the right lower quadrant. During a routine nursing check, he states that his "stomach" doesn't hurt anymore. The nurse should suspect that: 1) He is anxious about surgery. 2) He does not communicate effectively about pain. 3) His nausea and vomiting have decreased, thereby relieving his abdominal pain. 4) His appendix has ruptured.

1) He is anxious about surgery. Rationale: In the child with appendicitis, if the appendix perforates, the child will initially experience relief of pain. Other signs and symptoms will worsen, so that the child will appear acutely ill with high fever and signs of dehydration.

The parents of a 3 year old admitted for recurrent diarrhea are upset that the practitioner has not told them what is going on with their child. What is the priority intervention for this family? 1) Help the family develop a written list of specific questions to ask the practitioner. 2) Inform the family of the time that hospital rounds are made so that they can be present. 3) Immediately page the practitioner to come to the unit to speak with the family. 4) Answer all of the parents' questions about the child's illness.

1) Help the family develop a written list of specific questions to ask the practitioner. Rationale: Often families ask general questions of health care providers and do not receive the information they need. The nurse should determine what information the family does want and then help develop a list of questions. When the questions are written, the family can remember which questions to ask or can hand the sheet to the practitioner for answers. The nurse may have the information the parents want, but they are asking for specific information from the practitioner. Unless it is an emergency, the nurse should not place a stat page for the practitioner. Being present is not necessarily the issue but rather the ability to get answers to specific questions.

Iron overload is a side effect of chronic transfusion therapy. What treatment assists in minimizing this complication? 1) Infusion of deferoxamine 2) Hemoglobin electrophoresis 3) Magnetic therapy 4) Washing red blood cells (RBCs) to reduce iron

1) Infusion of deferoxamine Rationale: Deferoxamine infusions in combination with vitamin C allow the iron to remain in a more chelatable form. The iron can then be excreted more easily. Use of magnets does not remove additional iron from the body. Hemoglobin electrophoresis is used to confirm the diagnosis of hemoglobinopathies; it does not affect iron overload. Washed RBCs remove white blood cells and other proteins from the unit of blood; they do not affect the iron concentration.

Care for the child with acute idiopathic thrombocytopenic purpura (ITP) includes which therapeutic intervention? 1) Intravenous administration of anti-D antibody 2) Use of nonsteroidal anti-inflammatory drugs (NSAIDs) 3) Splenectomy 4) Helping child participate in sports

1) Intravenous administration of anti-D antibody Rationale: Anti-D antibody causes an increase in platelet count approximately 48 hr after administration. Splenectomy is reserved for chronic severe ITP not responsive to pharmacologic management. NSAIDs are not used in ITP. Both NSAIDs and aspirin interfere with platelet aggregation. The nurse works with the child and parents to choose quiet activities while the platelet count is below 100,000/mm3.

Sam, age 11, has a diagnosis of Rheumatic fever and has missed school for a week. What is the most likely cause of this problem? 1) Playing too much soccer and not getting enough rest 2) History of open heart surgery at 5 years of age 3) Previous streptococcal throat infection 4) Exposure to a sibling with pneumonia

3) Previous streptococcal throat infection Rationale: Rheumatic fever occurs as a sequela to group A streptococcal infection.

A 16-year-old adolescent male tells the school nurse that he is gay. The nurse's response is based on knowledge that: 1) It is important to provide a nonthreatening environment in which he can discuss this matter. 2) Homosexual adolescents do not have concerns that differ from heterosexual adolescents. 3) The nurse should feel open to discussing his or her own beliefs about homosexuality. 4) He is too young to have had enough sexual activity to determine this matter.

1) It is important to provide a nonthreatening environment in which he can discuss this matter. Rationale: Nurses who work with adolescents must develop communication skills that include assuring confidentiality, making no assumptions, remaining nonjudgmental, and posing open-ended questions. Although homosexual behavior in adolescence does not necessarily indicate that the adolescent will maintain a homosexual orientation, gay and lesbian adolescents face many challenges growing up in a society that is often unaccepting. Those adolescents who self-identify their sexual preference as homosexual during high school are at increased risk for a variety of health risks and problem behaviors, including suicide, victimization, risky sexual behaviors, and abuse of multiple substances. Nurses in schools and community clinics are in a position to provide guidance with appropriate information and referral in a confidential atmosphere. Nurses should strongly encourage adolescents to discuss sexuality, sexual behavior, and contraception with their parents whenever possible but must guarantee confidentiality of nurse-adolescent communication. The professional role of the nurse is to ensure that adolescents have the knowledge, skills, and opportunities to enable them to make responsible decisions about sexual behavior.

What immunization should not be given to a child receiving chemotherapy for cancer? 1) Measles, mumps, rubella (MMR) 2) Diphtheria, pertussis, tetanus (DPT) 3) Inactivated poliovirus vaccine 4) Tetanus vaccine

1) Measles, mumps, rubella (MMR) Rationale: The vaccine used for MMR is a live virus and can cause serious disease in immunocompromised children. The tetanus vaccine, inactivated poliovirus vaccine, and DPT are not live vaccines and can be given to immunosuppressed children. The immune response is likely to be suboptimum, so delaying vaccination is usually recommended.

What pain medication is contraindicated in children with sickle cell disease (SCD)? 1) Meperidine (Demerol) 2) Hydrocodone (Vicodin) 3) Morphine sulfate 4) Ketorolac (Toradol)

1) Meperidine (Demerol) Rationale: Meperidine (pethidine [Demerol]) is not recommended. Normeperidine, a metabolite of meperidine, is a central nervous system stimulant that produces anxiety, tremors, myoclonus, and generalized seizures when it accumulates with repetitive dosing. Patients with SCD are particularly at risk for normeperidine-induced seizures.

The nurse admits an infant with vomiting and the diagnosis of hypertrophic pyloric stenosis. Which metabolic alteration should the nurse plan to assess for with this infant? 1) Metabolic alkalosis 2) Respiratory alkalosis 3) Respiratory acidosis 4) Metabolic acidosis

1) Metabolic alkalosis Rationale: Laboratory findings may indicate metabolic alkalosis as a result of vomiting.

The nurse is conducting a teaching session for parents on nutrition. Which characteristics of families should the nurse consider that can cause families to struggle in providing adequate nutrition? (Select all that apply.) 1) Migrant status 2) Lower income 3) Homelessness 4) Single parent status 5) Working parents

1) Migrant status 2) Lower income 3) Homelessness

What are signs and symptoms of anemia? (Select all that apply.) 1) Pallor 2) Bradycardia 3) Dilute urine 4) Muscle weakness 5) Fatigue

1) Pallor 4) Muscle weakness 5) Fatigue

The nurse is completing a respiratory assessment on a newborn. What are normal findings of the assessment the nurse should document? (Select all that apply.) 1) Periodic breathing 2) Slight intercostal retractions 3) Respiratory rate of 40 breaths/min 4) Apnea lasting 25 seconds 5) Wheezes on auscultation

1) Periodic breathing 2) Slight intercostal retractions 3) Respiratory rate of 40 breaths/min Rationale: Periodic breathing is common in full-term newborns and consists of rapid, nonlabored respirations followed by pauses of less than 20 seconds. The newborn's respiratory rate is between 30 and 60 breaths/min. The ribs are flexible, and slight intercostal retractions are normal on inspiration. Periods of apnea lasting more than 20 seconds are abnormal, and wheezes should be reported.

The nurse is planning to admit a 14-year-old adolescent with Cushing syndrome. What clinical manifestations should the nurse expect to observe in this child? (Select all that apply.) 1) Petechial hemorrhage 2) Decreased pubic hair 3) Truncal obesity 4) Muscle weakness 5) Hyperpigmentation of elbows 6) Facial plethora

1) Petechial hemorrhage 3) Truncal obesity 6) Facial plethora Rationale: Clinical manifestations of Cushing syndrome include truncal obesity, petechial hemorrhage, and facial plethora. Decreased pubic and axillary hair; hyperpigmentation of elbows, knees, and wrists; and muscle weakness are clinical manifestations of adrenocortical insufficiency.

The nurse is caring for a child with persistent hypoxia secondary to a cardiac defect. The nurse recognizes the potential risk of a cerebrovascular accident (stroke). What strategy is an important objective to decrease this risk? 1) Prevent dehydration. 2) Reduce energy expenditure. 3) Minimize seizures. 4) Promote cardiac output.

1) Prevent dehydration. Rationale: In children with persistent hypoxia, polycythemia develops. Dehydration must be prevented in hypoxemic children because it potentiates the risk of strokes. Minimizing seizures, promoting cardiac output, and reducing energy expenditure will not reduce the risk of cerebrovascular accidents.

Which finding in the newborn is considered abnormal? 1) Profuse drooling 2) Slight vaginal reddish discharge 3) Nystagmus 4) Dark green or black stools

1) Profuse drooling Rationale: Profuse drooling and salivation are potential signs of a major abnormality. Newborns with esophageal atresia cannot swallow their oral secretions, resulting in excessive drooling. Nystagmus is an involuntary movement of the eyes. This is a common variation in newborns. Meconium, the first stool of newborns, is dark green or black. A pseudomenstruation may be present in normal newborns. This is a blood-tinged or mucoid vaginal discharge.

What is the major cause of death for children in the United States? 1) Heart disease 2) Childhood cancer 3) Congenital anomalies 4) Injuries

4) Injuries Unintentional injuries (accidents) are the leading cause of death after age 1 year through adolescence. The leading cause of death for those younger than 1 year is congenital anomalies, and childhood cancers and heart disease cause a significantly lower percentage of deaths in children older than 1 year of age.

What are some of the characteristics of systemic corticosteroid drugs? (Select all that apply.) 1) Prolonged use could suppress a child's growth. 2) Dosage should be gradually tapered. 3) Possess antimicrobial properties. 4) Effective vasodilator. 5) Inhibits inflammatory and allergic reactions.

1) Prolonged use could suppress a child's growth. 2) Dosage should be gradually tapered. 5) Inhibits inflammatory and allergic reactions. Rationale: Systemic corticosteroids are valuable in the treatment of severe skin disorders because of their capacity to inhibit inflammatory and allergic reactions. The dose is carefully adjusted and gradually tapered to the minimum dosage that is effective. Prolonged use may temporarily suppress the child's growth. Vasoconstriction is a possible effect of systemic steroids, and corticosteroids do not possess antimicrobial activity.

Which responsibilities are included in the pediatric nurse's promotion of the health and well-being of children? (Select all that apply.) 1) Providing support and counseling 2) Establishing lifelong friendships 3) Participating in ethical decision making 4) Promoting disease prevention 5) Establishing a therapeutic relationship 6) Providing financial assistance

1) Providing support and counseling 3) Participating in ethical decision making 4) Promoting disease prevention 5) Establishing a therapeutic relationship A pediatric nurse does not need to establish lifelong friendships or provide financial assistance to children and their families. Boundaries should be clearly defined

Which responsibilities are included in the pediatric nurse's promotion of the health and well-being of children? (Select all that apply.) 1) Providing support and counseling 2) Providing financial assistance 3) Establishing lifelong friendships 4) Participating in ethical decision making 5) Promoting disease prevention 6) Establishing a therapeutic relationship

1) Providing support and counseling 4) Participating in ethical decision making 5) Promoting disease prevention 6) Establishing a therapeutic relationship

The school nurse is assessing a child's severely scraped knee for infection. What are signs of a wound infection? (Select all that apply.) 1) Purulent exudate 2) Decreased temperature 3) Dry scab 4) Edema 5) Odor

1) Purulent exudate 4) Edema 5) Odor Rationale: Signs of wound infection are odor, edema, and purulent exudate. Increased, not decreased, temperature indicates infection. A dry scab over the wound is part of the healing process.

A child presents with a 2-day history of fever, abdominal pain, occasional vomiting, and decreased oral intake. Which finding would the nurse prioritize for immediate reporting to the provider? 1) Rebound tenderness and abdominal guarding 2) Child can tolerate only sips of fluid without nausea 3) Parents will be leaving the child alone in the hospital 4) Temperature 101.9° F

1) Rebound tenderness and abdominal guarding Rationale: Rebound pain and abdominal guarding are signs of an acute abdomen and should be reported immediately to the physician; surgery may be needed.

Which of the following are included in the evaluation step of the nursing process? (Select all that apply.) 1) Selecting alternative interventions if the outcome has not been met 2) Determination of whether or not the outcome has been met 3) Establishment of priorities and selecting expected patient goals 4) Determining if a risk or actual dysfunctional health problem exists 5) Ascertaining if the plan requires modification

1) Selecting alternative interventions if the outcome has not been met 2) Determination of whether or not the outcome has been met 5) Ascertaining if the plan requires modification Rationale: Evaluation is the last step in the nursing process. The nurse gathers, sorts, and analyzes data to determine whether (1) the established outcome has been met, (2) the nursing interventions were appropriate, (3) the plan requires modification, or (4) other alternatives should be considered. Establishing priorities and selecting expected patient goals are done in the outcomes identification/planning stage. Determining if a risk or actual dysfunctional health problem exists is done in the diagnosis stage of the nursing process. Nursing process: Assessment > Diagnosis > Outcomes Identification/Planning > Implementation > Evaluation

The nurse observes flaring of nares in a newborn. What should this be interpreted as? 1) Sign of respiratory distress 2) Snuffles of congenital syphilis 3) Nasal occlusion 4) Appropriate newborn breathing

1) Sign of respiratory distress Rationale: Nasal flaring is an indication of respiratory distress. A nasal occlusion should prevent the child from breathing through the nose. Because newborns are obligatory nose breathers, this should require immediate referral. Snuffles are indicated by a thick, bloody nasal discharge without sneezing. Sneezing and thin, white mucus drainage are common in newborns and are not related to nasal flaring.

The clinic nurse is assessing a child with bacterial conjunctivitis (pink eye). Which assessment findings should the nurse expect? (Select all that apply.) 1) Swollen eyelids 2) Crusting of eyelids in the morning 3) Inflamed conjunctiva 4) Itching 5) Purulent eye drainage

1) Swollen eyelids 2) Crusting of eyelids in the morning 3) Inflamed conjunctiva 5) Purulent eye drainage Rationale: The assessment findings in bacterial conjunctivitis include swollen eyelids, inflamed conjunctiva, purulent eye drainage, and crusting of eyelids in the morning. Itching is seen with allergic conjunctivitis but not with bacterial conjunctivitis.

An older school-age child asks the nurse, "What is the reason for this topical corticosteroid cream?" What rationale should the nurse give? 1) The cream is used for an anti-inflammatory effect. 2) The cream is used for an antibacterial effect. 3) The cream is used for an analgesic effect. 4) The cream is used for an antifungal effect.

1) The cream is used for an anti-inflammatory effect. Rationale: The glucocorticoids are the therapeutic agents used most widely for skin disorders. Their local anti-inflammatory effects are merely palliative, so the medication must be applied until the disease state undergoes a remission or the causative agent is eliminated. It does not have an antifungal, analgesic, or antibacterial effect.

An adolescent is being placed on a calcium channel blocker. What should the nurse inform the adolescent with regard to this medication? (Select all that apply.) 1) The medication may increase heart rate. 2) The medication may cause cold extremities. 3) The medication may cause fatigue. 4) The medication may cause peripheral edema. 5) The medication may cause constipation.

1) The medication may increase heart rate. 4) The medication may cause peripheral edema. 5) The medication may cause constipation.

A nurse is describing the endocrine system to a nursing student. Which statement best describes the negative feedback system of the pituitary gland? 1) The pituitary gland secretes hormones that stimulate target organs to produce specific hormones. As hormonal secretions of the target organs increase, the pituitary gland decreases the secretion of the stimulating hormone. 2) Target organs stimulate the hypothalamus, which in turn produces hormones. 3) The anterior pituitary stimulates the production of posterior pituitary hormones. 4) The pituitary gland produces five hormones that are interdependent.

1) The pituitary gland secretes hormones that stimulate target organs to produce specific hormones. As hormonal secretions of the target organs increase, the pituitary gland decreases the secretion of the stimulating hormone.

The nurse is precepting a new graduate nurse at an ambulatory pediatric hematology and oncology clinic. What cardinal signs of cancer in children should the nurse make the new nurse aware of? (Select all that apply.) 1) Unexplained loss of energy 2) Gradual, steady fever 3) Sudden tendency to bruise easily 4) Frequent headaches 5) Excessive, rapid weight gain 6) Transitory, generalized pain

1) Unexplained loss of energy 3) Sudden tendency to bruise easily 4) Frequent headaches Rationale: The cardinal signs of cancer in children include a sudden tendency to bruise easily; frequent headaches, often with vomiting; and an unexplained loss of energy. Other cardinal signs include persistent, localized pain; excessive, rapid weight loss; and a prolonged, unexplained fever.

The nurse is preparing an airborne infection isolation room for a patient. Which communicable disease does the patient likely have? 1) Varicella 2) Influenza 3) Pertussis 4) Scarlet fever

1) Varicella Rationale: An airborne infection isolation room is the isolation for persons with a suspected or confirmed airborne infectious disease transmitted by the airborne route such as measles, varicella, or tuberculosis. Pertussis, influenza, and scarlet fever require droplet transmission precautions.

A nurse is monitoring a child who has just had a tonsillectomy for signs of hemorrhage. Which of the following findings is a sign of this postoperative complication? 1) ​Frequent swallowing ​2) Reports of thirst ​3) Mouth breathing ​4) Reports of pain

1) ​Frequent swallowing Rationale: Immediately after surgery, the child should be assessed for bleeding and ability to swallow secretions. Postoperative hemorrhage is the most serious and life-threatening complication of tonsillectomy. The rate and quality of respirations and breath sounds should be assessed. Vital signs, including blood pressure, should be monitored frequently until discharge. The child is assessed for bleeding (frequent swallowing; restlessness; a fast, thready pulse; or vomiting bright red blood).

The nurse is teaching about skin care for atopic dermatitis. Which statement by the parent indicates that further teaching may be necessary? 1) "I will use Vaseline or Crisco to moisturize my child's skin." 2) "A hot bath will soothe my child's itching when it is severe." 3) "I will apply a small amount of the prescribed cream after the bath." 4) "I will buy cotton rather than wool or synthetic clothing for my child."

2) "A hot bath will soothe my child's itching when it is severe." Rationale: Hot baths should be avoided, as they are more dehydrating to the skin. Warm baths are preferred, followed by patting the child dry and rubbing on a small amount of prescribed cream, then a liberal amount of fragrance-free moisturizer. Wool clothing should be avoided in the child with atopic dermatitis.

A nurse in the emergency department is caring for a client who has epiglottitis. The child is crying, and the parents are concerned that their child has not had anything to eat or drink for several hours. The parents are also concerned about the other children catching this. Which of the following responses should the nurse give? 1) "The influenza vaccine can protect your other children from epiglottitis." 2) "Did you bring an item that will help comfort your child?" 3) "The nurse practitioner will be in shortly to obtain a throat culture." 4) "Your child can drink, but cannot have anything solid."

2) "Did you bring an item that will help comfort your child?" Rationale: The child with epiglottitis has an edematous epiglottis, which can completely obstruct the airway at any time. The danger of airway obstruction is so great that usually all invasive procedures, such as venipuncture, are postponed until the child is intubated. Throat and blood specimens are obtained for culture after the child is intubated. Because of the risk for aspiration, the child is kept on NPO status. The nurse should also keep the child as calm and quiet as possible. The nurse encourages parents of young children to have their children immunized against H. influenza to decrease the risk for contracting epiglottitis.

The nurse is caring for a 6-year-old child with acute lymphoblastic leukemia (ALL). The parent states, "My child has a low platelet count, and we are being discharged this afternoon. What do I need to do at home?" What statement is most appropriate for the nurse to make? 1) "You should give your child aspirin instead of acetaminophen for fever or pain." 2) "Your child should avoid contact sports or activities that could cause bleeding." 3) "You should feed your child a bland, soft, moist diet for the next week." 4) "Your child should avoid large groups of people for the next week."

2) "Your child should avoid contact sports or activities that could cause bleeding." Rationale: A child with a low platelet count needs to avoid activities that could cause bleeding such as playing contact sports, climbing trees, using playground equipment, or bike riding. The child should be given acetaminophen, not aspirin, for fever or pain; the child does not need to be on a soft, bland diet or avoid large groups of people because of the low platelet count.

Which vitamin supplementation has been found to reduce both morbidity and mortality in measles? 1) B12 2) A 3) Zinc 4) C

2) A Rationale: Evidence suggests that vitamin A supplementation reduces both morbidity and mortality in measles.

Which serious reaction should the nurse be alert for when administering vaccines? 1) Fever 2) Allergic reaction 3) Pain at injection site 4) Skin irritation

2) Allergic reaction Rationale: Each vaccine administration carries the risk of an allergic reaction. The nurse must be prepared to intervene if the child demonstrates signs of a severe reaction. Mild febrile reactions do occur after administration. The nurse includes management of fever in the parent teaching. Local skin irritation may occur at the injection site after administration. Parents are informed that this is expected. The injection can be painful. The nurse can minimize the discomfort with topical analgesics and nonpharmacologic measures.

The nurse is planning care for a hospitalized preschool-aged child. Which should the nurse plan, to ensure atraumatic care? 1) Ask that all family members leave the room when performing procedures 2) Allow the child to choose the type of juice to drink with the administration of oral medications 3) Explain that EMLA cream cannot be used for the morning lab draw because there is not enough time for it to be effective 4) Limit explanation of procedures because the child is preschool aged

2) Allow the child to choose the type of juice to drink with the administration of oral medications Allowing the child a choice of juice to drink when taking oral medications provides the child with a sense of control

A toddler, age 16 months, falls down a few stairs. He gets up and "scolds" the stairs as if they caused him to fall. What is this an example of? 1) Irreversibility 2) Animism 3) Delayed cognitive development 4) Ritualism

2) Animism Rationale: Animism is the attribution of lifelike qualities to inanimate objects. By scolding the stairs, the toddler is attributing human characteristics to them. Ritualism is the need to maintain sameness and reliability. It provides a sense of comfort to toddlers. Irreversibility is the inability to reverse or undo actions initiated physically. The toddler is acting in an age-appropriate manner.

The nurse is caring for an infant on the pediatric unit who has a very red rash in the diaper area, with red lesions scattered on the abdomen and thighs. What is the priority nursing intervention? 1) Institute contact isolation precautions 2) Apply topical antifungal cream 3) Apply topical antibiotic cream 4) Administer Griseofulvin with a fatty meal

2) Apply topical antifungal cream Rationale: An angry red rash with satellite lesions is typical of diaper candidiasis. Topical antifungal preparations are indicated.

The nurse is admitting a 7-year-old child to the pediatric unit for abdominal pain. To determine what the child understands about the reason for hospitalization, what should the nurse do? 1) Find out what the parents have told the child. 2) Ask the child why he came to the hospital today. 3) Question the parents about why they brought the child to the hospital. 4) Review the note from the admitting practitioner.

2) Ask the child why he came to the hospital today. Rationale: School-age children are able to answer questions. The only way for the nurse to know about the child's understanding of the reason for hospitalization is to ask the child directly. Finding out what the parents told the child and why they brought the child to the hospital or reading the admitting practitioner's description of the reason for admission will not provide information about what the child has heard and retained.

An adolescent patient wants to make decisions about treatment options, along with his parents. Which moral value is the nurse displaying when supporting the adolescent to make decisions? 1) Beneficence 2) Autonomy 3) Nonmaleficence 4) Justice

2) Autonomy

The nurse begins CPR on a 5-year-old unresponsive client. When the emergency response team arrives, the child continues to have no respiratory effort but has a heart rate of 50 with cyanotic legs. What should the team do next? 1) Discontinue compressions, but continue administering breaths with a bag-mask device. 2) Begin 2-person CPR at a ratio of 2 breaths to 15 compressions. 3) Begin 2-person CPR at a ratio of 2 breaths to 30 compressions. 4) Establish an intravenous line with a large bore needle while preparing the defibrillator.

2) Begin 2-person CPR at a ratio of 2 breaths to 15 compressions. Rationale: CPR is done on children for heart rate of less than 60 with signs of poor perfusion. Rescuers should use 15:2 compression to ventilation ratio for 2 rescuer CPR for a child. Breaths without compressions are indicated only for respiratory arrests where the heart rate remains above 60. The AED/defibrillator should be used as soon as it is ready, but rescuers should not discontinue compressions until the device is ready for use. The ratio for 2-person CPR in adults is 30:2.

A 3-year-old child is to receive a medication that is supplied as an enteric-coated tablet. What is the best nursing action? 1) Crush the tablet and mix it with apple sauce. 2) Check with the prescriber to see if an alternative form can be used. 3) Dissolve the medication in the child's milk. 4) Place a pill in the posterior part of the pharynx and tell the child to swallow.

2) Check with the prescriber to see if an alternative form can be used. Rationale: A 3-year-old would typically have difficulty swallowing tablets, pills, or capsules. Therefore, the nurse should check with the prescriber to see if an alternative form, such as a liquid, is available. If not, then a different medication may need to be ordered. Enteric-coated medication should not be crushed or dissolved because this would interfere with the action of the drug. No medication should ever be mixed with an essential food for the child.

The nurse is talking to a group of parents of school-age children at an after school program about childhood health problems. Which statement should the nurse include in the teaching? 1) Immunization rates are the same among children of different races and ethnicities. 2) Childhood obesity is the most common nutritional problem among children. 3) Dental caries are not a problem commonly seen in children since the introduction of fluorinated water. 4) Mental health problems are typically not seen in school-age children, but may be diagnosed in adolescence

2) Childhood obesity is the most common nutritional problem among children.

The clinic nurse is evaluating causes for iron deficiency due to impaired iron absorption. What should the nurse recognize as causes for iron deficiency due to impaired iron absorption? (Select all that apply.) 1) Absence of phosphates 2) Chronic diarrhea 3) Gastric acidity 4) Inflammatory bowel disease 5) Lactose intolerance

2) Chronic diarrhea 4) Inflammatory bowel disease 5) Lactose intolerance

An 8-year-old girl is receiving a blood transfusion when the nurse notes that she has developed precordial pain, dyspnea, distended neck veins, slight cyanosis, and a dry cough. These manifestations are most suggestive of what complication? 1) Hemolytic reaction 2) Circulatory overload 3) Air embolism 4) Allergic reaction

2) Circulatory overload Rationale: The signs of circulatory overload include distended neck veins, hypertension, crackles, a dry cough, cyanosis, and precordial pain. Signs of air embolism are sudden difficulty breathing, sharp pain in the chest, and apprehension. Urticaria, pruritus, flushing, asthmatic wheezing, and laryngeal edema are signs and symptoms of allergic reactions. Hemolytic reactions are characterized by chills, shaking, fever, pain at infusion site, nausea, vomiting, tightness in chest, flank pain, red or black urine, and progressive signs of shock and renal failure.

What finding is a clinical manifestation of increased intracranial pressure (ICP) in children? 1) Sunken fontanel 2) Diplopia, blurred vision 3) Low-pitched cry 4) Increased blood pressure

2) Diplopia, blurred vision Rationale: Diplopia and blurred vision are signs of increased ICP in children. A high-pitched cry and a tense or bulging fontanel are characteristic of increased ICP. Increased blood pressure, common in adults, is rarely seen in children.

When would a child diagnosed with insulin-dependent diabetes mellitus most likely demonstrate a decreased need for insulin? 1) During adolescence 2) During the "honeymoon" phase 3) During minor illnesses 4) During growth spurts

2) During the "honeymoon" phase Rationale: After initiation of insulin therapy, the child may have a "honeymoon" phase characterized by hypoglycemia and a decreasing need for insulin. This may last from a few weeks to 1 year or longer.

At a visit to the pediatric clinic, a mother is concerned by her 4-year-old's symptoms over the last few weeks. Which of the following symptoms described by the mother would lead the nurse to be concerned about an oncologic disorder? (Select all that apply.) 1) Bruising in various stages, mainly on the legs 2) Enlarged, firm lymph nodes 3) Frequent complaints of respiratory infections, while siblings remain healthy 4) Asthma symptoms with increase in wheezing 5) Fever for more than 1 week

2) Enlarged, firm lymph nodes 3) Frequent complaints of respiratory infections, while siblings remain healthy 5) Fever for more than 1 week Rationale: Bruising of varying healing stages on the lower extremities is common in children who are active - crawling, running, jumping. Bruising from an oncologic cause can occur without any source of trauma. May see petechia, or pinpoint bruises that indicate abnormal bleeding and lack of clotting factors. Asthma symptoms with increased wheezing doesn't necessarily indicate a concern for cancer. This may be a normal finding in a child with an asthma exacerbation. Common to have increased wheezing on occasion related to excursion, allergies, or lack of adherence to treatment plan. Alterations in immune function, firm/enlarged lymph nodes, fever lasting longer than a week are all reason to suspect an oncologic disorder.

Which are components of the FLACC scale? (Select all that apply.) 1) Capillary refill time 2) Facial expression 3) Color 4) Leg position 5) Activity

2) Facial expression 4) Leg position 5) Activity Rationale: Facial expression, consolability, cry, activity, and leg position are components of the FLACC scale. Color is a component of the Apgar scoring system. Capillary refill time is a physiologic measure that is not a component of the FLACC scale.

Parents of a hospitalized toddler ask the nurse, "What is meant by family-centered care?" the nurse should respond with which statement? 1) Family-centered care avoids expecting families to be part of the decision-making. 2) Family-centered care recognizes that the family is the constant in a child's life. 3) Family-centered care encourages family dependence on the health care system. 4) Family-centered care reduces the effect of cultural diversity of the family

2) Family-centered care recognizes that the family is the constant in a child's life.

Which is the best nursing action when a child with insulin-dependent diabetes mellitus is sweating, trembling, and pale? 1) Offer the child a glass of water. 2) Give the child a glass of orange juice. 3) Give the child glucagon subcutaneously. 4) Give the child 5 units of Regular insulin subcutaneously.

2) Give the child a glass of orange juice. Rationale: The child is exhibiting signs and symptoms of hypoglycemia. Hypoglycemia should be treated immediately with oral consumption of 15 g of easily digested (simple) carbohydrates. In 15 minutes, if symptoms are not relieved or blood glucose is 80 mg/dL or lower, repeat treatment. If the hypoglycemia occurs during the night, treat with 30 g of carbohydrate (1/2 simple and ½ complex) and with protein. Early treatment reduces the possibility of a more severe reaction. There are 15 g of carbohydrates in 4 oz of 100% fruit juice.

The nurse is reviewing the HEALTHY People 2020 leading health indicators for a child health promotion program. Which are included in the leading health indicators? (Select all that apply.) 1) Reduce incidences of cancer 2) Improve immunization rates 3) Decrease tobacco use 4) Decrease the incidence of eating disorders 5) Increase access to health care

2) Improve immunization rates 3) Decrease tobacco use 5) Increase access to health care The Healthy People 2020 leading health indicators provide a framework for identifying essential components for child health promotion programs designed to prevent future health problems in our nation's children. Some of the leading health indicators include decreasing tobacco use, improving immunization rates, and increasing access to health care. Reducing the incidence of cancer and decreasing the incidence of eating disorders are not on the list as leading health indicators.

The nurse is teaching parents about the visual ability of their newborn. Which should the nurse include in the teaching session? (Select all that apply.) 1) Tear glands do not begin to function until 8 to 12 weeks of age. 2) Infants can momentarily fixate on a bright object that is within 8 inches. 3) The infant prefers bright colors (red, orange, blue) over medium colors (yellow, green, pink). 4) Visual acuity is between 20/100 and 20/400. 5) The infant demonstrates visual preferences of black-and-white contrasting patterns.

2) Infants can momentarily fixate on a bright object that is within 8 inches. 4) Visual acuity is between 20/100 and 20/400. 5) The infant demonstrates visual preferences of black-and-white contrasting patterns. Rationale: Visual acuity is reported to be between 20/100 and 20/400, depending on the vision measurement techniques. The infant has the ability to momentarily fixate on a bright or moving object that is within 20 cm (8 inches) and in the midline of the visual field. The infant demonstrates visual preferences of black-and-white contrasting patterns. The visual preference is for medium colors (yellow, green, pink) over dim or bright colors (red, orange, blue). Tear glands begin to function until 2 to 4 weeks of age.

What is a clinical finding that warrants further intervention for the child with acute poststreptococcal glomerulonephritis? 1) Weight loss to within 1 pound of the pre-illness weight 2) Inspiratory crackles 3) Urine output of 1 milliliter per kilogram per 4) A normal blood pressure

2) Inspiratory crackles Rationale: Inspiratory crackles may be an indicator of a life-threatening complication of glomerulonephritis called pulmonary edema. Return to pre-illness weight, urine output of 1 milliliter per kilogram per, and a normal blood pressure are indicative or resolution of illness.

When assessing a 2-year-old child at the clinic for a routine check-up, which skill should the nurse expect the child to be able to perform? 1) Riding a tricycle 2) Kicking a ball forward 3) Tying his or her shoelaces 4) Using blunt scissors

2) Kicking a ball forward Rationale: A 2-year-old child usually can kick a ball forward. Riding a tricycle is characteristic of a 3-year-old child. Tying shoelaces is a behavior to be expected of a 5-year-old child. Using blunt scissors is a characteristic of a 3-year-old child.

A 17-year-old adolescent tells the nurse that he is not having sex because it would make his parents very angry. This response indicates that the adolescent has a developmental lag in which area? 1) Cognitive development 2) Moral development 3) Psychosexual development 4) Psychosocial development

2) Moral development Rationale: During stage 3 of Kohlberg's Theory of Moral Development, morality is based pm avoiding disapproval or disturbing the conscience; the child is becoming socially sensitive and is motivated by the conformity to please other and avoid punishment. The teenager's sense of justice is developed through interpersonal relationships with peers, family, and other adult role models.

The nurse is doing a neurologic assessment on a 2-month-old infant after a car accident. Moro, tonic neck, and withdrawal reflexes are present. How should the nurse interpret these findings? 1) Decerebrate posturing 2) Neurologic health 3) Decorticate posturing 4) Severe brain damage

2) Neurologic health Rationale: Moro, tonic neck, and withdrawal reflexes are three reflexes that are present in a healthy 2-month-old infant and are expected in this age-group.

The nurse is caring for a 5-year-old child with a congenital heart anomaly causing chronic cyanosis. When performing the history and physical examination, what is the nurse least likely to assess? 1) Clubbing of the nail beds 2) Obesity from overeating 3) Exercise intolerance 4) Squatting during play activities

2) Obesity from overeating Rationale: Children with CHD causing chronic cyanosis are likely to demonstrate failure to thrive, not obesity. They frequently develop clubbing of the nail beds and exercise intolerance, and those with Tetralogy of Fallot or pulmonary stenosis may display cyanotic spells (squatting).

When performing cardiopulmonary resuscitation (CPR), which finding indicates that external chest compressions are effective? 1) Mottling of the skin 2) Palpable pulse 3) Visible waveform on the cardio-respiratory monitor 4) ETCO2 reading of 5

2) Palpable pulse

When discussing discipline with the mother of a 4-year-old child, which should the nurse include? 1) Withdrawal of love and approval is effective at this age. 2) Parental control should be consistent. 3) Children as young as 4 years rarely need to be disciplined. 4) One should expect rules to be followed rigidly and unquestioningly.

2) Parental control should be consistent. Rationale: For effective discipline, parents must be consistent and must follow through with agreed-on actions. Withdrawal of love and approval is never appropriate or effective. The 4-year-old child will test limits and may misbehave. Children of this age do not respond to verbal reasoning. Realistic goals should be set for this age-group. Discipline is necessary to reinforce these goals. Discipline strategies should be appropriate to the child's age and temperament and the severity of the misbehavior. Following rules rigidly and unquestioningly is beyond the developmental capabilities of a 4-year-old child.

The parents of a young child with heart failure (HF) tell the nurse that they are nervous about giving digoxin. The nurse's response should be based on which knowledge? 1) It is difficult to either overmedicate or undermedicate with digoxin. 2) Parents need to learn specific, important guidelines for administration of digoxin. 3) Parents lack the expertise necessary to administer digoxin. 4) It is a safe, frequently used drug.

2) Parents need to learn specific, important guidelines for administration of digoxin. Rationale: Digoxin has a narrow therapeutic range. The margin of safety between therapeutic, toxic, and lethal doses is very small. Specific guidelines are available for parents to learn how to administer the drug safely and to monitor for side effects. Parents may lack the expertise to administer the drug at first, but with discharge preparation, they should be prepared to administer the drug safely.

The nurse is preparing to complete documentation in a patient's chart. Which should be included in the documentation of nursing care? (Select all that apply.) 1) Incident reports 2) Patient's response to provision of care 3) Initial assessments 4) Nursing care provided 5) Reassessments

2) Patient's response to provision of care 3) Initial assessments 4) Nursing care provided 5) Reassessments

A 3-month-old infant presents with a history of vomiting after feeding. The plan for the infant is to rule out GER. What information from the history would lead the nurse to believe that this infant may need further intervention? 1) Small "spits" after feeding 2) Poor weight gain 3) Sleeps through the night 4) Difficult to burp

2) Poor weight gain Rationale: GER is considered a routine and benign occurrence unless it is significant enough to cause respiratory symptoms or, as in this infant, to interfere with growth, in which case it would be considered gastroesophageal reflux disease, and would warrant treatment.

The nurse is aware that which age-group is at risk for childhood injury because of the cognitive characteristic of magical and egocentric thinking? 1) Adolescent 2) Preschool 3) Young school age 4) Middle school age

2) Preschool Rationale: Preschool children have the cognitive characteristic of magical and egocentric thinking, meaning they are unable to comprehend danger to self or others. Young and middle school-aged children have transitional cognitive processes, and they may attempt dangerous acts without detailed planning but recognize danger to themselves or others. Adolescents have formal operational cognitive processes and are preoccupied with abstract thinking

When administering ear drops to a 2-year-old, which action would be most appropriate? 1) Massage the child's forehead to facilitate absorption of the medication. 2) Pull the pinna of the child's ear down and back. 3) Have the child turn his head to the opposite side after giving the drops. 4) Tell the child that the drops are to treat his infection

2) Pull the pinna of the child's ear down and back. Rationale: When administering ear drops to a child under age 3, the nurse should pull the pinna of the ear down and back. The nurse should explain the reason for the ear drops to the child at a level that the child can understand, and it is doubtful that a 2-year-old would understand the term "infection." Once the drops are given, the child should remain in the same position for several minutes to ensure that the medication remains in the ear canal. The nurse should massage the area anterior to the affected ear to facilitate passage of the medication into the ear canal.

A child with heart failure is on Lanoxin (digoxin). The laboratory value a nurse must closely monitor is which? 1) Serum chloride 2) Serum potassium 3) Serum glucose 4) Serum sodium

2) Serum potassium Rationale: A fall in the serum potassium level enhances the effects of digoxin, increasing the risk of digoxin toxicity. Increased serum potassium levels diminish digoxin's effect. Therefore, serum potassium levels (normal range, 3.5 to 5.5 mmol/L) must be carefully monitored.

The mother asks the nurse for advice about discipline for her 18-month-old. Which discipline strategy should the nurse suggest that the mother use? 1) Reprimand 2) Time-out 3) Reasoning 4) Spanking

2) Time-out Rationale: Time-out is the most appropriate discipline for toddlers. It helps to remove them from the situation and allows them to regain control. Structuring interactions with 3-year-olds helps minimize unacceptable behavior. This approach involves setting clear and reasonable rules and calling attention to unacceptable behavior as soon as it occurs. Reprimanding a young child can reinforce undesirable behavior over time because it provides attention. Physical punishment, such as spanking, has limited effectiveness and serious negative effects. Reasoning is more appropriate for older children, such as preschoolers and those older, especially when moral issues are involved. Unfortunately, reasoning combined with scolding often takes the form of shame or criticism, and children take such remarks seriously, believing that they are "bad".

Which finding indicates that a child receiving prednisone for primary nephrotic syndrome is in remission? 1) Urine is negative for casts for 5 days 2) Urine is 0 to trace for protein for 5 to 7 days 3) Urine is 0 to trace for blood for 1 week 4) Urine is negative for protein for 2 weeks

2) Urine is 0 to trace for protein for 5 to 7 days Rationale: Remission is defined as zero to trace urine protein for 3 to 7 consecutive days. Some children respond to steroids quickly and achieve remission in 5 to 7 days, whereas others may not respond for 4 weeks.

The nurse is assessing breath sounds on a child. Which are expected auscultated breath sounds? (Select all that apply.) 1) Wheezes 2) Vesicular 3) Bronchial 4) Crackles 5) Bronchovesicular

2) Vesicular 3) Bronchial 5) Bronchovesicular

A child with hypoparathyroidism is receiving vitamin D therapy. The parents should be advised to watch for which signs or symptoms of vitamin D toxicity? 1) Anorexia and insomnia 2) Weakness and lassitude 3) Headache and seizures 4) Physical restlessness, voracious appetite without weight gain

2) Weakness and lassitude Rationale: Vitamin D toxicity can be a serious consequence of therapy. Parents are advised to watch for weakness, fatigue, lassitude, headache, nausea, vomiting, and diarrhea. Renal impairment is manifested through polyuria, polydipsia, and nocturia. Headaches may be a sign of vitamin D toxicity, but seizures are not. Anorexia and insomnia are not characteristic of vitamin D toxicity. Physical restlessness and a voracious appetite with weight loss are manifestations of hyperthyroidism.

Which stool characteristic should the nurse expect to assess with a child diagnosed with intussusception? 1) Hard stools positive for guaiac 2) Loose, foul-smelling stools 3) "Currant jelly" stools 4) Ribbon-like stools

3) "Currant jelly" stools Rationale: A hallmark of Hirschsprung Disease is constipation with passage of foul-smelling, ribbon-like stools. "Currant jelly" stools is a hallmark of intussusception. Foul-smelling, fatty stools is indicative of malabsorption or steatorrhea. Hard stools that are positive for blood during guaiac testing are associated with constipation.

Which statement made by a 14-year-old adolescent who is newly diagnosed with insulin-dependent diabetes mellitus (IDDM) indicates a need for further teaching? 1) "Exercise will decrease my insulin requirements." 2) "I need to check my sugars before meals and at bedtime." 3) "It is okay to drink chocolate milk with meals." 4) "I should eat meals and snack at the same time every day."

3) "It is okay to drink chocolate milk with meals." Rationale: The goal of nutrition is to promote normal growth and development, encourage healthy nutrition, prevent complications, and maintain near-normal blood glucose levels. Because the insulin dosage is balanced with food intake, the diet plan should stress a consistent intake, particularly of carbohydrate food products. Both the timing of the meal or snack and the amount of food are important in avoiding hyperglycemia and hypoglycemia. Adherence to a daily schedule that maintains a consistent food intake combined with consistent insulin injections aids in achieving metabolic control.

A parent of a toilet-trained 3-year-old expresses concern over her child's bed-wetting while hospitalized. What should the nurse tell the parent? 1) "This is normal. Your child probably received too much fluid the night before." 2) "Your child was too immature to be toilet trained. In a few months, your child should be old enough." 3) "It is very common for children to regress when they are in the hospital." 4) "Children are afraid in the hospital and frequently wet their bed."

3) "It is very common for children to regress when they are in the hospital." Rationale: A child will regress to a behavior used in an earlier stage of development in order to cope with a perceived threatening situation. Readiness for toilet training should be based on neurological, physical, and psychological development, not the age of the child. Children are afraid of hospitalization, but bed-wetting is a compensatory mechanism done to regress to a previous stage of development that is more comfortable and secure for the child. Telling the mother that bed-wetting is related to fluid intake does not provide an adequate explanation for the underlying regression.

A nurse is teaching an assistive personnel to count the respiratory rate of a newborn. Which of the following statements indicates understanding of why the respiratory rate should be counted for a complete minute? 1) "Newborns do not expand their lungs fully with each respiration." 2) "Activity will increase the respiration rate." 3) "The rate and rhythm are irregular in newborns." 4) "Newborns are abdominal breathers."

3) "The rate and rhythm are irregular in newborns." Rationale: Newborn breathing is very irregular. To obtain an accurate measure of respiratory rate, count respirations for one full minute.

An adolescent is scheduled for a leg amputation in 2 days for treatment of osteosarcoma. What approach should the nurse implement? 1) Explain that although the amputation is difficult, it will cure the cancer. 2) Help the adolescent accept the amputation as better than a long course of chemotherapy. 3) Answer questions with straightforward honesty. 4) Avoid discussing the seriousness of the condition.

3) Answer questions with straightforward honesty. Rationale: Honesty is essential to gain the child's cooperation and trust. The diagnosis of cancer should not be disguised with falsehoods. The adolescent should be prepared for the surgery, so there is time for reflection about the diagnosis and subsequent treatment. This allows questions to be answered. To accept the need for radical surgery, the child must be aware of the lack of alternatives for treatment. Amputation is necessary, but it will not guarantee a cure. Chemotherapy is an integral part of the therapy with surgery. The child should be informed of the need for chemotherapy and its side effects before surgery.

A nurse is caring for a child who is in a plaster spica cast. Which of the following actions should the nurse take? 1) Avoid turning the child until the cast is dry. 2) Assist the patient with crutch walking after the cast is dry. 3) Apply moleskin to the edges of the cast. 4) Use a heat lamp to facilitate drying.

3) Apply moleskin to the edges of the cast. Rationale: The nurse should apply moleskin to the edges of the cast to prevent the cast from rubbing on the patient's skin. A cool fan can be used to facilitate drying of a plaster cast. The child should be turned every 2 hours to expose all areas of the cast to facilitate drying. A patient who has a spica cast is non-weight bearing until the cast is removed.

A child with a nut allergy presents with a severe reaction for the third time in 3 months. The parent says, "I am having trouble with the food labels." What should the nurse do first? 1) Refer the client to the dietician. 2) Obtain a social service consult. 3) Assess the parent's ability to read. 4) Notify the health care provider (HCP).

3) Assess the parent's ability to read. Rationale: Three severe reaction sin 3 months indicate a serious problem with adhering to the prevention plan. The nurse should first determine if the parent can actually read the label. The underlying problem may be that the parent is visually impaired or unable to read. The parent's reading level determines what additional support is needed. Referrals to social service or dietary may be indicated, but the nurse does not yet have enough information about the problem. The nurse would communicate with the HCP after assessing the situation to recommend referrals.

The nurse is assessing the reflexes of a newborn. Stroking the outer sole of the foot assesses which reflex? 1) Dance or step 2) Perez 3) Babinski 4) Grasp

3) Babinski Rationale: This is a description of the Babinski reflex. Stroking the outer sole of the foot upward from the heel across the ball of the foot causes the big toes to dorsiflex and the other toes to hyperextend. This reflex persists until approximately age 1 year or when the newborn begins to walk. The grasp reflex is elicited by touching the palms or soles at the base of the digits. The digits will flex or grasp. The Perez reflex involves stroking the newborn's back when prone; the child flexes the extremities, elevating the head and pelvis. This disappears at ages 4 to 6 months. When the newborn is held so that the sole of the foot touches a hard surface, there is a reciprocal flexion and extension of the leg, simulating walking. This reflex disappears by ages 3 to 4 weeks.

The nurse is preparing a community outreach program about the prevention of iron-deficiency anemia in infants. What statement should the nurse include in the program? 1) Iron-fortified cereal should be introduced to the infant at 2 months of age. 2) Whole milk can be introduced into the infant's diet in small amounts at 6 months. 3) Breast milk or iron-fortified formula should be used for the first 12 months. 4) Iron supplements cannot be given until the infant is older than 1 year of age.

3) Breast milk or iron-fortified formula should be used for the first 12 months. Rationale: Prevention, the primary goal in iron-deficiency anemia, is achieved through optimal nutrition and appropriate iron supplements. The American Academy of Pediatrics recommends feeding an infant only breast milk or iron-fortified formula for the first 12 months of life. Whole cow's milk should not be introduced until after 12 months, iron supplements can be given during the first year of life, and iron-fortified cereals should not be introduced until the infant is 4 to 6 months old.

The nurse is caring for a newborn with a large ventricular septal defect. The client has undergone pulmonary artery banding. Which assessment findings indicate that the pulmonary artery band is functioning effectively? 1) Radial pulses are bounding 2) Urine output is greater than 1 mL/kg/h 3) Breath sounds are clear and equal bilaterally 4) Capillary refill is less than 2 seconds

3) Breath sounds are clear and equal bilaterally Rationale: Pulmonary artery banding is a tx used in pediatric clients with congenital cardiac defects with increased pulmonary blood flow. The pulmonary artery band reduces excessive pulmonary blood flow and protects the lungs from irreversible damage. When the pulmonary artery band is functioning properly, the lungs should no longer be receiving an increased amount of blood flow, which would be reflected in clear and equal breath sounds. A capillary refill less than 3 seconds and a urine output greater than 1 ml/kg/h reflect adequate peripheral perfusion. Bounding radial pulses suggest increased pulmonary blood flow.

Which assessment finding should the nurse expect in an infant with Hirschsprung disease? 1) Diarrhea 2) Foul-smelling, fatty stools 3) Constipation with passage of foul-smelling, ribbon-like stools 4) "Currant jelly" stools

3) Constipation with passage of foul-smelling, ribbon-like stools Rationale: A hallmark of Hirschsprung Disease is constipation with passage of foul-smelling, ribbon-like stools. "Currant jelly" stools is a hallmark of intussusception. Foul-smelling, fatty stools is indicative of malabsorption or steatorrhea. Diarrhea, increase in the frequency of stools or decrease in the consistency of stool content, is not associated with Hirschsprung Disease.

Which best describes signs and symptoms as part of a nursing diagnosis? 1) Human response to state of illness or health 2) Description of potential risk factors 3) Cues and clusters derived from patient assessment 4) Identification of actual health problems

3) Cues and clusters derived from patient assessment S&S are the cues and clusters of defining characteristics that are derived from a patient assessment and indicate actual health problems

A nurse is caring for a toddler admitted with laryngotracheobronchitis who is placed in a cool mist tent. As a result of this treatment, the nurse expects to observe: 1) Decrease in fever 2) Barking cough 3) Decreased stridor 4) Improved hydration

3) Decreased stridor Rationale: The goal of treatment is to maintain a patent airway. The use of a cool mist may relieve mucosal swelling which would be reflected in a decrease in stridor and barking cough.

After a generalized seizure, what symptoms should the nurse expect the child to experience? 1) Irritability and hunger 2) Nervousness and excitability 3) Diarrhea and abdominal discomfort 4) Lethargy and confusion

3) Diarrhea and abdominal discomfort Rationale: In the postictal phase, after a generalized seizure, the child may remain semiconscious and difficult to arouse. The average duration of the postictal phase is usually 30 minutes. The child may remain confused or sleep for several hours. He or she may have mild impairment of fine motor movements. The child may have visual and speech difficulties and may vomit or complain of headache.

The parents of a preschooler reports that the child creates a scene every night at bedtime. What is the best course of action? 1) Allow the child to stay up later one or two nights a week. 2) Encourage active play before bedtime. 3) Establish a set bedtime and follow a routine. 4) Give the child a cookie if bedtime is pleasant.

3) Establish a set bedtime and follow a routine. Rationale: Bedtime is often a problem with preschoolers. Recommendations for reducing conflicts at bedtime include establishing a routine, such as story reading, and conveying the expectation that the child will comply. Allowing the child to stay up late one or two nights interferes with establishing the needed bedtime rituals. Excitement, such as active play, just before bedtime should be avoided because it stimulates the child, making it difficult for the child to calm down and prepare for sleep. Using food as a reward if bedtime is pleasant should be avoided because it places too much importance on food. Other rewards, such as stickers, could be used as an alternative.

The mother of a 3-month-old breastfed infant asks about giving her baby water because it is summer and very warm. What should the nurse tell her? 1) Clear juices are better than water to promote adequate fluid intake. 2) Water once or twice a day will make up for losses resulting from environmental temperature. 3) Fluids in addition to breast milk are not needed. 4) Water should be given if the infant seems to nurse longer than usual.

3) Fluids in addition to breast milk are not needed. Rationale: Infants who are breastfed or bottle-fed do not need additional water during the first 4 months of life. Excessive intake of water can create problems such as water intoxication, hyponatremia, or failure to thrive. Juices provide empty calories for infants.

The nurse is assessing heart sounds on a school-age child. Which should the nurse document as abnormal findings if found on the assessment? (Select all that apply.) 1) S1 louder at the apex of the heart 2) S3 heart sound 3) Grade II murmur 4) S4 heart sound 5) S2 louder than S1 in the aortic area

3) Grade II murmur 4) S4 heart sound 5) S2 louder than S1 in the aortic area

What nutritional component should be altered in the infant with heart failure (HF)? 1) Decrease in fats 2) Decrease in protein 3) Increase in calories 4) Increase in fluids

3) Increase in calories Rationale: Infants with HF have a greater metabolic rate because of poor cardiac function and increased heart and respiratory rates. Their caloric needs are greater than those of average infants, yet their ability to take in calories is diminished by their fatigue. The diet should include increased protein and increased fat to facilitate the child's intake of sufficient calories. Fluids must be carefully monitored because of the HF.

A 13-year-old child with cystic fibrosis (CF) is a frequent patient on the pediatric unit. This admission, she is sleeping during the daytime and unable to sleep at night. What should be a beneficial strategy for this child? 1) Administer prescribed sedative at night to aid in sleep. 2) Arrange a consult with the social worker to determine whether issues at home are interfering with her care. 3) Negotiate a daily schedule that incorporates hospital routine, therapy, and free time. 4) Have the practitioner speak with the child about the need for rest when receiving therapy for CF.

3) Negotiate a daily schedule that incorporates hospital routine, therapy, and free time. Rationale: Children's response to the disruption of routine during hospitalization is demonstrated in eating, sleeping, and other activities of daily living. The lack of structure is allowing the child to sleep during the day, rather than at night. Most likely the lack of schedule is the problem. The nurse and child can plan a schedule that incorporates all necessary activities, including medications, mealtimes, homework, and patient care procedures. The schedule can then be posted, so the child has a ready reference. Sedatives are not usually used with children. The child has a chronic illness and most likely knows the importance of rest. The parents and child can be questioned about changes at home since the last hospitalization.

The nurse is closely monitoring a child who is unconscious after a fall and notices that the child suddenly has a fixed and dilated pupil. How should the nurse interpret this? 1) Eye trauma 2) Severe brain stem damage 3) Neurosurgical emergency 4) Brain death

3) Neurosurgical emergency Rationale: The sudden appearance of a fixed and dilated pupil(s) is a neurosurgical emergency. The nurse should immediately report this finding. Although a dilated pupil may be associated with eye trauma, this child has experienced a neurologic insult. One fixed and dilated pupil is not suggestive of brain death. Pinpoint pupils or fixed, bilateral pupils for more than 5 minutes are indicative of brain stem damage. The unilateral fixed and dilated pupil is suggestive of damage on the same side of the brain.

The clinical manifestations of sickle cell anemia are primarily a result of which pathologic process? 1) Bone marrow failure in which the formed elements of the blood are distressed 2) Basic deficit of factor VIII 3) Obstructions caused by abnormal red blood cells and increased destruction of red blood cells 4) Diffuse fibrin deposition in the microvasculature causing occlusions

3) Obstructions caused by abnormal red blood cells and increased destruction of red blood cells

Which factor contributes to early adolescents engaging in risk-taking behaviors? 1) A desire to master their environment 2) Engagement in the process of separation from their parents 3) Peer pressure 4) A belief that they are invulnerable

3) Peer pressure Rationale: Impulsivity, inexperience, and peer pressure may place adolescents in unsafe situations. Feelings of invulnerability ("It can't happen to me") persist, and little thought may be given to the negative consequences of certain behaviors.

A client is in the emergency department with sneezing and coughing. The client is in the triage area, waiting to be seen by a health care provider. To prevent spread of infection to others in the area and to the health care staff, what should the nurse do? 1) Ask the client to wash the hands before being examined. 2) Give the client a surgical mask to wear. 3) Place the client in an isolation room. 4) Ask the others in the area to move away from the client.

3) Place the client in an isolation room. Rationale: In order to prevent infections in hospitals, the nurse institutes measures to contain respiratory secretions in symptomatic clients. The nurse gives the client a mask to wear and tissues; the nurse instructs the client to dispose of used tissues in a no-touch receptacle. It is not necessary to place the client in isolation. It is not appropriate to ask others to move away from the client, but the nurse can ask the client to keep 3 feet away from others in the waiting room, if there is room. The nurse instructs the client to perform hand hygiene after blowing his nose or touching his nose or face, but doing so is not a prerequisite for being examined by the HCP. The nurse and HCP also use hand hygiene practices when caring for this client.

What should the nurse include in a teaching plan for the parents of a child with vesicoureteral reflux? 1) Suggestions for how to maintain fluid restrictions 2) The need for the child to hold urine for 6 to 8 hours 3) Screening for urinary tract infection (UTI) if febrile 4) The use of bubble baths as an incentive to increase bath time

3) Screening for urinary tract infection (UTI) if febrile Rationale: Children with VUR are often given prophylactic antibiotics and screened for UTI every 2 to 4 months and when febrile. Urinary stasis and performance of bubble baths are risk factors that place the child at an increased risk for the development of a UTI. A primary goal in the therapeutic management of VUR is to prevent infection. Fluid restrictions are not necessary to implement when caring for the child with VUR.

A 5-month-old infant with RSV bronchiolitis is in respiratory distress. The infant has copious secretions, increased work of breathing, cyanosis, and a respiratory rate of 78. what is the most appropriate initial nursing intervention? 1) Bring the emergency equipment to the room and begin bag-valve-mask ventilation. 2) Attempt to calm the infant by placing him in his mother's lap and offering him a bottle. 3) Suction secretions, provide 100% oxygen via mask, and anticipate respiratory failure. 4) Alert the physician or nurse practitioner to the situation and ask for an order for a stat chest radiograph.

3) Suction secretions, provide 100% oxygen via mask, and anticipate respiratory failure. Rationale: Priorities of care for the child with respiratory distress are to clear the airway and provide oxygen supplementation. Children who experience respiratory distress often deteriorate very quickly, and the nurse must be prepared in the event of respiratory failure or arrest.

A 17-month-old child should be expected to be in which stage, according to Piaget? 1) Secondary circular reactions 2) Concrete operations 3) Tertiary circular reactions 4) Preoperations

3) Tertiary circular reactions Rationale: A 17-month-old child is in the fifth stage of the sensorimotor phase, tertiary circular reactions. The child uses active experimentation to achieve previously unattainable goals. Preoperations are the stage of cognitive development usually present in older toddlers and preschoolers. Concrete operations are the cognitive stage associated with school-age children. The secondary circular reaction stage lasts from about ages 4 to 8 months.

The nurse is preparing to administer a measles, mumps, rubella, and varicella (MMRV) vaccine. Which is a contraindication associated with administering this vaccine? 1) The child has recently been exposed to an infectious disease. 2) The child is having intermittent episodes of diarrhea. 3) The child has a disorder that causes a deficient immune system. 4) The child has symptoms of a cold but no fever.

3) The child has a disorder that causes a deficient immune system. Rationale: The MMRV (measles, mumps, rubella, and varicella) vaccine is an attenuated live virus vaccine. Children with deficient immune systems should not receive the MMRV vaccine because of a lack of evidence of its safety in this population. Exposure to an infectious disease, symptoms of a cold, or intermittent episodes of diarrhea are not contraindications to receiving a live vaccine.

The parents of a 3-year-old child with congenital heart disease are afraid to let their child play with other children because of possible overexertion. How should the nurse reply to this concern? 1) The parents should meet all the child's needs. 2) The child needs to understand that peers' activities are too strenuous. 3) The child needs opportunities to play with peers. 4) Constant parental supervision is needed to avoid overexertion.

3) The child needs opportunities to play with peers. Rationale: The child needs opportunities for social development. Children are able to regulate and limit their activities based on their energy level. Parents must be encouraged to seek appropriate social activities for the child, especially before kindergarten. The child needs to have activities that foster independence.

A Hispanic toddler has pneumonia. The nurse notices that the parent consistently feeds the child only the broth that comes on the clear liquid tray. Food items, such as Jell-O, Popsicles, and juices, are left. Which statement best explains this? 1) The parent is trying to feed the child only what the child likes most. 2) Hispanics believe an innate energy called chi is strengthened by eating soup. 3) The parent is trying to restore normal balance through appropriate "hot" remedies. 4) Hispanics believe the "evil eye" enters when a person gets cold

3) The parent is trying to restore normal balance through appropriate "hot" remedies. In several cultures, including Filipino, Chinese, Arabic, and Hispanic, hot and cold describe certain properties completely unrelated to temperature. Respiratory conditions such as pneumonia are "cold" conditions and are treated with "hot" foods

Which statement reflects accurate information about patterns of sleep and wakefulness in the newborn? 1) States of sleep are independent of environmental stimuli. 2) Cycles of sleep states are uniform in newborns of the same age. 3) The quiet alert stage is the best stage for newborn stimulation. 4) Muscle twitches and irregular breathing are common during deep sleep.

3) The quiet alert stage is the best stage for newborn stimulation. Rationale: During the quiet alert stage, the newborn's eyes are wide open and bright. The newborn responds to the environment by active body movement and staring at close-range objects. Newborns' ability to control their own cycles depend on their neurobehavioral development. Each newborn has an individual cycle. Muscle twitches and irregular breathing are common during light sleep.

A mother who breastfeeds her 6-week-old infant every 4 hr tells the nurse that he seems "hungry all the time." The nurse should recommend: 1) supplemental formula. 2) no change in feedings. 3) more frequent feedings. 4) newborn cereal.

3) more frequent feedings. Rationale: Infants who are breastfed tend to be hungry every 2 to 3 hr. They should be fed frequently. Six weeks is too early to introduce newborn cereal. Supplemental formula is not indicated. Giving additional formula or water to a breastfed infant may satiate the infant and create problems with breastfeeding. The infant requires additional feedings. Four hours is too long between feedings for a breastfed infant.

Which statement made by a school-age girl indicates the need for further teaching about the prevention of urinary tract infections? 1) "I always wear cotton underwear." 2) "I drink four to six glasses of fluid every day." 3) "I go to the bathroom every 3 to 4 hours." 4) "I really enjoy taking a bubble bath."

4) "I really enjoy taking a bubble bath." Rationale: UTI preventative measures includes use of cotton underwear, rather than synthetic fabric. Bubble baths, which can irritate the urinary system, should be avoided. Scheduled toileting is not required; however, it is recommended that toilet-trained children avoid "holding" urine and urinate at least four times per day, emptying the bladder completely.

The nurse is teaching a nursing student about standard precautions. Which statement made by the student indicates a need for further teaching? 1) "I will use precautions when I change an infant's diaper." 2) "I will use precautions when I encounter blood and body fluids." 3) "I will use precautions when I give an infant oral care." 4) "I will use precautions when administering oral medications to a school-age child."

4) "I will use precautions when administering oral medications to a school-age child." Rationale: Standard precautions involve the use of barrier protection (personal protective equipment [PPE]), such as gloves, goggles, a gown, or a mask, to prevent contamination from (1) blood, (2) all body fluids, secretions, and excretions except sweat, regardless of whether they contain visible blood, (3) nonintact skin, and (4) mucous membranes. Precautions should be taken when giving oral care, when changing diapers, and when encountering blood and body fluids. Further teaching is needed if the student indicates the need to use precautions when administering an oral medication to a school-age child.

A nurse is caring for a toddler who is diagnosed with hip dysplasia and has been placed in a hip spica cast. The child's mother asks the nurse why a Pavlik harness is not being used. Which of the following responses should the nurse make? 1) "The Pavlik harness is used for school-age children." 2) "The Pavlik harness is used for children with scoliosis, not hip dysplasia." 3) "The Pavlik harness cannot be used for your child because her condition is too severe." 4) "The Pavlik harness is used for infants less than 6 months of age."

4) "The Pavlik harness is used for infants less than 6 months of age." Rationale: The Pavlik harness is a soft brace designed for infants less than 6 months of age. A toddler is too large to fit into the brace.

What is the best age to introduce solid food into an infant's diet? 1) When tooth eruption has started 2) When birth weight has tripled 3) 2 to 3 months 4) 4 to 6 months

4) 4 to 6 months Rationale: Physiologically and developmentally, 4- to 6-month-old infants are in a transition period. The extrusion reflex has disappeared, and swallowing is a more coordinated process. In addition, the gastrointestinal tract has matured sufficiently to handle more complex nutrients and is less sensitive to potentially allergenic food. Infants of this age will try to help during feeding. Two to three months is too young. The extrusion reflex is strong, and the child will push food out with the tongue. No research indicates that the addition of solid food to a bottle has any benefit. Infant birth weight doubles at 1 year. Solid foods can be started earlier. Tooth eruption can facilitate biting and chewing; most infant foods do not require this ability.

When developing a teaching plan about illness for the parent of a preschooler, which information should the nurse include about how a preschooler perceives illness? 1) A necessary part of life 2) The will of God 3) A test of self-worth 4) A punishment for wrongdoing

4) A punishment for wrongdoing Rationale: Preschool-age children may view illness as punishment for their fantasies. At this age, children do not have the cognitive ability to separate fantasies from reality and may expect to be punished for their "evil thoughts". Viewing illness as a necessary part of life requires a higher level of cognition than preschoolers possess. This view is seen in children of middle-school age and older. Perceiving illness as a test of self-worth or as the will of God is more characteristic of adults.

At a well-child visit, hydrocephalus may be suspected in an infant if upon assessment the nurse finds: 1) Sunken fontanels 2) Narrow sutures 3) Increase in weight since last visit 4) A rapid increase in head circumference

4) A rapid increase in head circumference Rationale: In the infant, the most obvious indication of hydrocephalus is often a rapid increase in head circumference. Assessment may also reveal bulging, tense fontanels with widening sutures.

What information should the nurse include when teaching the mother of a 9-month-old infant about administering liquid iron preparations? 1) Allow preparation to mix with saliva and bathe the teeth before swallowing. 2) Stop immediately if nausea and vomiting occur. 3) Give with meals. 4) Adequate dosage will turn the stools a tarry green color.

4) Adequate dosage will turn the stools a tarry green color. Rationale: The nurse should prepare the mother for the anticipated change in the child's stools. If the iron dose is adequate, the stools will become a tarry green color. A lack of color change may indicate insufficient iron. The iron should be given in two divided doses between meals when the presence of free hydrochloric acid is greatest. Iron is absorbed best in an acidic environment. Vomiting and diarrhea may occur with iron administration. If these occur, the iron should be given with meals, and the dosage reduced and gradually increased as the child develops tolerance. Liquid preparations of iron stain the teeth; they should be administered through a straw and the mouth rinsed after administration.

What is an appropriate nursing intervention when caring for a child in traction? 1) Keeping child prone to maintain good alignment 2) Removing adhesive traction straps daily to prevent skin breakdown 3) Providing active range of motion exercises to affected extremity three times a day 4) Assessing for tightness, weakness, or contractures in uninvolved joints and muscles

4) Assessing for tightness, weakness, or contractures in uninvolved joints and muscles Rationale: Traction places stress on the affected bone, joint, and muscles. The nurse must assess for tightness, weakness, or contractures developing in the uninvolved joints and muscles. The adhesive straps should be released or replaced only when absolutely necessary. Active, passive, or active with resistance exercises should be carried out for the unaffected extremity only. Movement is expected with children. Each time the child moves, the nurse should check to ensure that proper alignment is maintained.

Which is the most consistent and commonly used data for assessment of pain in infants? 1) Self-report 2) Physiologic 3) Parental report 4) Behavioral

4) Behavioral Rationale: Behavioral assessment is useful for measuring pain in young children and preverbal children who do not have the language skills to communicate that they are in pain. Infants are not able to self-report. Physiologic measures are not able to distinguish between physical responses to pain and other forms of stress. Parental report without a structured tool may not accurately reflect the degree of discomfort.

A child with hemophilia fell while riding his bicycle. He was wearing a helmet and did not lose consciousness. He has a mild abrasion on his knee that is not oozing. He is complaining of abdominal pain. What is the priority nursing assessment? 1) Perform neurologic checks 2) Examine his knee frequently 3) Assess ability to void frequently 4) Carefully assess his abdomen

4) Carefully assess his abdomen Rationale: The child's complaint of abdominal pain indicates that undetected bleeding may be present in the abdomen. Determining whether internal bleeding is present would take priority over the knee abrasion, which has nearly stopped bleeding.

The nurse is caring for a child receiving a continuous intravenous (IV) low-dose infusion of morphine for severe postoperative pain. The nurse observes a slower respiratory rate, and the child cannot be aroused. The most appropriate management of this child is for the nurse to do which first? 1) Direct the charge nurse to call a Code Blue. 2) Document clinical findings. 3) Discontinue morphine until the child is fully awake. 4) Discontinue infusion and administer naloxone (Narcan)

4) Discontinue infusion and administer naloxone (Narcan) Rationale: The management of opioid-induced respiratory depression includes lowering the rate of infusion and stimulating the child. If the respiratory rate is depressed and the child cannot be aroused, then IV naloxone should be administered. The child will be in pain because of the reversal of the morphine. The morphine should be discontinued, but naloxone is indicated if the child is unresponsive.

Which muscle is contraindicated for the administration of immunizations in infants and young children? 1) Ventrogluteal 2) Anterolateral thigh 3) Deltoid 4) Dorsogluteal

4) Dorsogluteal Rationale: The dorsogluteal site is avoided in children because of the location of nerves and veins. The deltoid is recommended for 12 months and older. The ventrogluteal and anterolateral thigh sites can safely be used for the administration of vaccines to infants.

Which situation denotes a nontherapeutic nurse-patient-family relationship? 1) The nurse is planning to read a favorite fairy tale to a patient. 2) The nurse is discussing with a fellow nurse the emotional draw to a certain patient. 3) The nurse is working with a family to find ways to decrease the family's dependence on health care providers. 4) During shift report, the nurse is criticizing parents for not visiting their child.

4) During shift report, the nurse is criticizing parents for not visiting their child. Rationale: Criticizing parents for not visiting in shift report is nontherapeutic and shows an under involvement with the parents. Reading a fairy tale is a therapeutic and age appropriate action. Discussing feelings of an emotional draw with a fellow nurse is therapeutic and shows a willingness to understand feelings. Working with parents to decrease dependence on health care providers is therapeutic and helps to empower the family.

A child is receiving propylthiouracil for the treatment of hyperthyroidism (Graves disease). The parents and child should be taught to recognize and report which sign or symptom immediately? 1) Upper respiratory tract infection 2) Fatigue 3) Weight loss 4) Fever, sore throat

4) Fever, sore throat Rationale: Children being treated with propylthiouracil must be carefully monitored for the side effects of the drug. Parents must be alerted that sore throat and fever accompany the grave complication of leukopenia. These symptoms should be immediately reported. Fatigue and weight loss are manifestations of hyperthyroidism. Their presence may indicate that the drug is not effective but does not require immediate evaluation. Upper respiratory tract infections are most likely viral in origin and not a sign of leukopenia.

A cancer patient is experiencing neuropathic cancer pain. Which prescription should the nurse expect to be ordered? 1) Hydromorphone (Dilaudid) 2) Lorazepam (Ativan) 3) Morphine sulfate (MS Contin) 4) Gabapentin (Neurontin)

4) Gabapentin (Neurontin) Rationale: Anticonvulsants (gabapentin, carbamazepine) have demonstrated effectiveness in neuropathic cancer pain. Ativan is an antianxiety agent, and Dilaudid and MS Contin are opioid analgesics.

The mother of a child who was recently diagnosed with acute glomerulonephritis asks the nurse why the physician keeps talking about "casts" in the urine. The nurse's response is based on the knowledge that the presence of casts in the urine indicates: 1) Recent streptococcal infection 2) Excessive amounts of protein in the urine 3) Glomerular healing 4) Glomerular injury

4) Glomerular injury Rationale: Casts originate from the renal tubules. Presence of WBC casts in urine (pyuria) indicates infection or inflammation of the GU system. The presence of RBC casts are an indicator of renal disease.

The nurse is preparing a child for possible alopecia from chemotherapy. What information should the nurse include? 1) Regrowth of hair usually begins 12 months after chemotherapy ends. 2) Wearing hats or scarves is preferable to a wig. 3) Expose head to sunlight to stimulate hair regrowth. 4) Hair may have a slightly different color or texture when it regrows.

4) Hair may have a slightly different color or texture when it regrows. Rationale: Alopecia is a side effect of certain chemotherapeutic agents and cranial irradiation. When the hair regrows, it may be of a different color or texture. Children should choose the head covering they prefer. A wig should be selected similar to the child's own hairstyle and color before the hair loss. The head should be protected from sunlight to avoid sunburn. The hair usually grows back within 3 to 6 months after the cessation of treatment.

The nurse conducts a wellness screening on a 9-year-old client. Which finding most suggests that the client has typical social development? 1) Enjoys active play 2) Is able to organize and plan 3) Thinks independently 4) Has a best friend

4) Has a best friend Rationale: During the school-age years, children learn to socialize with children of the same age. The "best friend" stage, which occurs around 9 or 10 years of ae, is important in providing a foundation for self-esteem and later relationships. Thinking independently, organizing, and planning are cognitive skills. Active play relates to motor skills.

Which nursing diagnosis has the highest priority for the child with celiac disease? 1) Altered growth and development related to obesity 2) Fluid volume excess related to celiac crisis 3) Pain related to chronic constipation 4) Imbalanced nutrition: Less than body requirements related to malabsorption

4) Imbalanced nutrition: Less than body requirements related to malabsorption Rationale: The major manifestations in the child with celiac disease include diarrhea and growth failure. Celiac disease requires dietary modification to prevent chronic maldigestion and malabsorption.

Which statement indicates the best sequence for the nurse to conduct an assessment in a nonemergency situation? 1) Perform the physical examination and then ask the family if there are any problems in the child's life. 2) Get a complete history of the family's health beliefs and practices, and then assess the child. 3) Do the physical examination while at the same time asking about the child's previous illnesses; then talk about the family's concerns. 4) Introduce yourself, ask about any problems, take a history, and do the physical examination.

4) Introduce yourself, ask about any problems, take a history, and do the physical examination. Rationale: The nurse will have the information to do a better physical examination if he or she first determines any significant history or problems.

Because of the possible teratogenic effects, it is imperative the nurse provides education to the female adolescent of childbearing age when which medication is prescribed? 1) Azelaic acid 2) Clindamycin 3) Tretinoin 4) Isotretinoin

4) Isotretinoin Rationale: The most serious side effect of isotretinoin is the potential teratogenic effects. Isotretinoin is absolutely contraindicated in pregnant women. Sexually active young women must use an effective contraceptive method during treatment and for 1 month after treatment. Clindamycin, azelaic acid, and tretinoin are all topical agents.

The nurse is assessing a child who was just admitted to the hospital for observation after a head injury. What clinical manifestation is the most essential part of the nursing assessment to detect early signs of a worsening condition? 1) Posturing 2) Focal neurologic signs 3) Vital signs 4) Level of consciousness

4) Level of consciousness Rationale: The most important nursing observation is assessment of the child's level of consciousness. Alterations in consciousness appear earlier in the progression of an injury than do alterations of vital signs or focal neurologic signs. Neurologic posturing is indicative of neurologic damage.

The school nurse is assessing children for risk factors related to childhood injuries. Which child has the most risk factors related to childhood injury? 1) Female, multiple siblings, stable home life 2) Male, even tempered, history of previous injuries 3) Female, reacts negatively to new situations, no serious previous injuries 4) Male, high activity level, stressful home life

4) Male, high activity level, stressful home life

The parent tells the nurse that an 8-year-old child is continually telling jokes and riddles to the point of driving the other family members crazy. The nurse should explain this behavior is a sign of which factor? 1) Inappropriate peer influence 2) Excessive television watching 3) Inadequate parental attention 4) Mastery of language ambiguities

4) Mastery of language ambiguities Rationale: School-age children delight in riddles and jokes. Mastery of the ambiguities of language and of sentence structures allows the school-age child to manipulate words, and telling riddles and joes is a way of practicing this skill. Children who suffer from inadequate attention from parents tend to demonstrate abnormal behavior. Peer influence is less important to school-age children, and while the child may learn the joke from a friend, he or she is telling the joke to master language. Watching television does not influence the extent of joke telling.

The nurse notices that a child is increasingly apprehensive and has tachycardia after heart surgery. The chest tube drainage is now 8 mL/kg/hr. What should be the nurse's initial intervention? 1) Apply warming blankets. 2) Encourage child to cough, turn, and deep breathe. 3) Give additional pain medication per protocol. 4) Notify the practitioner of these findings.

4) Notify the practitioner of these findings. Rationale: The practitioner is notified immediately. Increases of chest tube drainage to more than 3 mL/kg/hr for more than 3 consecutive hours or 5 to 10 mL/kg in any 1 hr may indicate postoperative hemorrhage. Increased chest tube drainage with apprehensiveness and tachycardia may indicate cardiac tamponade—blood or fluid in the pericardial space constricting the heart—which is a life-threatening complication. Warming blankets are not indicated at this time. Additional pain medication can be given before the practitioner drains the fluid, but the notification is the first action. Encouraging the child to cough, turn, and deep breathe should be deferred until after evaluation by the practitioner.

Which food choice by a parent of a 2-year-old child with celiac disease indicates a need for further teaching? 1) Meat patty 2) Rice cake 3) Corn 4) Oatmeal

4) Oatmeal Rationale: Children with celiac disease should avoid the consumption of wheat, rye, barley, and oats due to their gluten allergy.

A nurse is teaching an adolescent about Tanner stages. Which statement best describes Tanner staging? 1) Staging of puberty based on the initiation of menarche and nocturnal emissions 2) Staging of puberty based on the initiation of primary sexual characteristics 3) Predictable stages of puberty that are based on chronological age 4) Predictable stages of puberty that are based on primary and secondary sexual characteristics

4) Predictable stages of puberty that are based on primary and secondary sexual characteristics Rationale: The Tanner scale (also known as the Tanner stages or Sexual Maturity Rating [SMR]) is a scale of physical development in children, adolescents, and adults. The scale defines physical measurements of development based on external primary and secondary sex characteristics, such as breast and pubic hair development in females and genital and pubic hair development in males.

What rationale explains why prolonged use of oxygen should be discouraged in a child with anemia? 1) Prolonged use of oxygen can interfere with iron production. 2) Prolonged use of oxygen interferes with a child's appetite. 3) Prolonged use of oxygen can affect the synthesis of hemoglobin. 4) Prolonged use of oxygen can decrease erythropoiesis.

4) Prolonged use of oxygen can decrease erythropoiesis. Rationale: Oxygen administration is of limited value, because each gram of hemoglobin is able to carry a limited amount of the gas. In addition, prolonged use of supplemental oxygen can decrease erythropoiesis. Prolonged use of oxygen does not interfere with iron production, a child's appetite, or affect the synthesis of hemoglobin.

Which diagnostic finding is assessed by the nurse when a child has primary nephrotic syndrome? 1) Leukocytosis 2) Hyperalbuminemia 3) Positive ASO titer 4) Proteinuria

4) Proteinuria Rationale: Nephrotic syndrome is a kidney disorder characterized by proteinuria, hypoalbuminemia, and edema. A positive antistreptolysin (ASO) titer indicates the presence of antibodies to streptococcal bacteria. The ASO titer would be expected to be negative in the child with nephrotic syndrome, but positive in the child with glomerulonephritis. Leukocytosis is not a primary finding, but may be present if disease is caused by an underlying system disease.

The nurse uses the five Ps to assess ischemia in a child with a fracture. What finding is considered a late and ominous sign? 1) Posturing 2) Positioning 3) Petaling 4) Pulselessness

4) Pulselessness Rationale: Pulselessness is an ominous sign that requires immediate notification of the practitioner. Permanent muscle and tissue damage can occur within 6 hr. The other signs of ischemia that need to be reported are pain, pallor, pulselessness, and paralysis. Petaling is a method of placing protective or smooth edges on a cast. Posturing is not a sign of peripheral ischemia. Finding a position of comfort can be difficult with a fracture. It would not be an ominous sign unless pain was increasing or uncontrollable.

Which is appropriate language development for an 8-month-old? The child should be: 1) Saying "dada" and "mama" specifically ("dada" to father and "mama" to mother). 2) Saying "ball" when parents point to a ball. 3) Saying three other words besides "mama" and "dada". 4) Saying "dada" and "mama" nonspecifically.

4) Saying "dada" and "mama" nonspecifically. Rationale: It is important for the nurse to assist parents in assessing speech development in their child so that developmental delays can be identified early. According to the Denver Developmental Screening Examination, at 7 to 8 months of age, the child should say "mama" and "dada" nonspecifically and imitate speech and sounds. Children cannot say "dada" or "mama" specifically or use more than three words until they are about 10 to 12 months of age. A child cannot respond to specific commands or point to objects when requested until about 15 to 17 months of age.

The nurse is explaining different parenting styles to a group of parents. The nurse explains that an authoritative parenting style can lead to which child behavior? 1) Submissiveness 2) Shyness 3) Self-consciousness 4) Self-reliance

4) Self-reliance

The diagnosis of hypertension depends on accurate assessment of blood pressure (BP). What is the appropriate technique to measure a child's BP? 1) Use a narrow cuff to ensure that the readings are correct. 2) Assess BP while the child is standing. 3) Compare left arm with left leg BP readings. 4) Serial measurements with child in sitting position with feet on the floor.

4) Serial measurements with child in sitting position with feet on the floor. Rationale: The diagnosis of hypertension is made after the BP is elevated on three separate occasions. Take the BP in a quiet area with the appropriate size cuff and the child sitting. Although left arm and left leg BP readings may be compared, it is not the procedure to diagnose hypertension. The appropriate size cuff is indicated. The most common cause of inaccurate readings is the use of a cuff that is too small.

The nurse assessing a child with acute post-streptococcal glomerulonephritis should be alert for which finding? 1) Hypotension 2) Weight loss 3) Increased urine output 4) Tea-colored urine

4) Tea-colored urine Rationale: Gross hematuria in the patient with glomerulonephritis may present as tea colored, cola-colored, or dirty green colored urine (indicates kidney injury), accompanied by hypertension and edema, and weight gain as a result of edema. The acute phase, or initial onset of the illness, can be accompanied by decreased urinary output.

What is the most important information to be included in the discharge planning for an infant with gastroesophageal reflux? 1) Reinforce the parent's knowledge of the infant's developmental needs. 2) Teach the parents to position the infant on the left side. 3) Have the parents keep an accurate record of intake and output. 4) Teach the parents how to do infant cardiopulmonary resuscitation (CPR).

4) Teach the parents how to do infant cardiopulmonary resuscitation (CPR). Rationale: Family education and support are vital. Training for parents in infant CPR is mandatory. While the other selections are recommended and necessary to complete, CPR training takes precedence due to the risk for apnea or bradycardia associated with GERD.

A 6-year-old child has patient-controlled analgesia (PCA) for pain management after orthopedic surgery. The parents are worried that their child will be in pain. What should your explanation to the parents include? 1) The child will continue to sleep and be pain free. 2) There is a high risk of overdose, so monitoring is done every 15 minutes. 3) Parents cannot administer additional medication with the button. 4) The pump can deliver baseline and bolus dosages.

4) The pump can deliver baseline and bolus dosages. Rationale: The PCA prescription can be set for a basal rate for a continuous infusion of pain medication. Additional doses can be administered by the patient, parent, or nurse as necessary. Although the goal of PCA is to have effective pain relief, a pain-free state may not be possible. With a 6-year-old child, the parents and nurse must assess the child to ensure that adequate medication is being given because the child may not understand the concept of pushing a button. Evidence-based practice suggests that effective analgesia can be obtained with the parents and nurse giving boluses as necessary. The prescription for the PCA includes how much medication can be given in a defined period. Monitoring every 1 to 2 hr for patient response is sufficient.

Which is an accurate description of homosexual (or gay-lesbian) families? 1) The stability needed to raise healthy children is lacking. 2) A nurturing environment is lacking. 3) The children become homosexual like their parents. 4) The quality of parenting is equivalent to that of nongay parents.

4) The quality of parenting is equivalent to that of nongay parents.

A child with secondary enuresis who complains of dysuria or urgency should be evaluated for which condition? 1) Glomerulonephritis 2) Nephrotic syndrome 3) Hypocalciuria 4) Urinary tract infection

4) Urinary tract infection Rationale: Manifestations of UTI in children include abdominal or suprapubic pain, voiding frequency, voiding urgency, dysuria, new or increased incidence of enuresis, and fever. Hypocalciuria or low calcium in the urine is observed in conditions such as eclampsia in pregnancy. Hematuria, proteinuria, edema, and signs of renal insufficiency would be observed with glomerulonephritis. Proteinuria, hypoalbuminemia, and edema would be observed in the patient with nephrotic syndrome.

Which intervention is appropriate when examining a male infant for cryptorchidism? 1) Taking a rectal temperature 2) Placing the infant on the examination table 3) Cooling the examiner's hands 4) Warming the room

4) Warming the room Rationale: Tips for assessing for cryptorchidism include examining the infant in a warm environment, ensuring that the infant is calm, warming your hands before touching the infant. Testes can retract into the inguinal canal if the infant is upset or cold. The cremasteric reflex, testicular retraction in response to tactile stimulation to the front inner thigh, can lead to false diagnosis of cryptorchidism.

A child, age 10 years, has a neuroblastoma and is in the hospital for additional chemotherapy treatments. What laboratory values are most likely this child's? 1) Platelets, 450,000/mm3; hemoglobin, 12 g/dL 2) White blood cell count, 17,000/mm3; hemoglobin, 15 g/dL 3) White blood cell count, 10,000/mm3; platelets, 175,000/mm3 4) White blood cell count, 3,000/mm3; hemoglobin, 11.5 g/dL

4) White blood cell count, 3,000/mm3; hemoglobin, 11.5 g/dL Rationale: Chemotherapy is the mainstay of therapy for extensive local or disseminated neuroblastoma. The drugs of choice are vincristine, doxorubicin, cyclophosphamide, cisplatin, etoposide, ifosfamide, and carboplatin. These cause immunosuppression, so the laboratory values will indicate a low white blood cell count and hemoglobin.

A nurse in a post-anesthesia recovery unit is admitting a client who is postoperative following a tonsillectomy. Which of the following actions should the nurse take to prevent aspiration? 1) Perform chest physiotherapy (CPT). 2) Suction the nasopharynx as needed. 3) Place a bedside humidifier at the head of the client's bed. 4) Withhold fluids until the client demonstrates a gag reflex.

4) Withhold fluids until the client demonstrates a gag reflex. Rationale: Immediately after surgery, the child should be assessed for bleeding and ability to swallow secretions. Suction equipment should be available, but do not suction unless there is airway obstruction. The use of CPT and/or a humidifier is not indicated in the care of the post-operative tonsillectomy patient.

When planning care for adolescents, the nurse should: 1) teach parents first, and they, in turn, will teach the teenager. 2) provide information for their long-term health needs because teenagers respond best to long-range planning. 3) maintain the parents' role by providing explanations for treatment and procedures to the parents only. 4) give information privately to adolescents about how they can manage the specific problems that they identify.

4) give information privately to adolescents about how they can manage the specific problems that they identify. Rationale: During well visits for health promotion, adolescents confer privately with the nurse and the health provider; separately, parents are asked about any concerns they might have. The professional role of the nurse is to ensure that adolescents have the knowledge, skills, and opportunities to enable them to make responsible decisions about their health care.

An infant with respiratory syncytial virus (RSV) is being admitted to the hospital. The nurse should stress the importance of: 1) the use of a respirator 2) bloodborne disease precautions 3) enteric precautions 4) hygiene/cough etiquette

4) hygiene/cough etiquette Rationale: Respiratory hygiene/cough etiquette stresses the importance of source control measures to contain respiratory secretions to prevent droplet and fomite transmission of viral respiratory tract infections.


Kaugnay na mga set ng pag-aaral

Accounting- Chapter 20 True/False

View Set

NURS 200 - CH. 12 Global Nutrition

View Set

Pharm 2 Chapters 27, 30, 37, and 38

View Set